IBPS Clerk 2015 Prelims Previous Year Paper

IBPS Clerk 2015 Prelims Paper 2

Section

Questions

Marks

English

30 Questions (1 – 30)

30

Logical Reasoning

30 Questions

30

Quantitative Aptitude

15 Questions

15

Data Interpretation

5 Questions (96 – 100)

5

Questions: 1 – 5

Rearrange the following five sentences (A), (B), (C), (D) and (E) in a proper sequence so as to form a meaningful paragraph and then answer the given question given below :

A. She thought it was very dangerous to have that mallet there, & started talking to herself.

B. It must have been there for a long time but somehow she had never noticed it before, & she began thinking.

C. Saying all this, she threw her tools, sat herself down & began crying.

D. Once evening when Mia had gone down to the cellar, she happened to look up at the ceiling & saw a mallet (a kind of wooden hammer) stuck in one of the beams.

E. She said “Suppose I was to be married, & have a son, & he was to grow up to be a man, & come down into the cellar, like I am doing now & the mallet was to fall on his head & kill him, what a dreadful thing it would be!”

 

Q. 1 Which of the following should be the LAST (FIFTH) sentence after the rearrangement ?

A. A

B. E

C. C

D. D

E. B

 

Q. 2 Which of the following should be the FIRST sentence after the rearrangement ?

A. A

B. B

C. C

D. D

E. E

 

Q. 3 Which of the following should be the THIRD sentence after the rearrangement ?

A. A

B. B

C. C

D. D

E. E

 

Q. 4 Which of the following should be the SECOND sentence after the rearrangement ?

A. A

B. B

C. C

D. D

E. E

 

Q. 5 Which of the following should be the FOURTH sentence after the rearrangement ?

A. A

B. B

C. C

D. D

E. E

 

Questions: 6 – 10

In these questions, sentences with four quoted words are given. One from four words given in the quotes may be either wrongly spelt or inappropriate in the context of the sentence. Find out the word which is wrongly spelt or inappropriate, if any. That word will be your answer. If all words given in the quotes are correctly spelt and also appropriate in the context of the sentence, then ‘All correct’ is your answer.

 

Q. 6 The government has “permitted” foreign direct investment in “digitail service” providers upto a limit of 50 “percent”.

A. permitted

B. digitail

C. service

D. percent

E. All correct

 

Q. 7 A bank gift card is “usually” a “prepaid” card which can be used in an ATM to “cheque” the balance but one cannot “withdraw” cash.

A. usually

B. prepaid

C. cheque

D. withdraw

E. All correct

 

Q. 8 He first “joinned” the Board of Directors two years “ago” & has expressed his concerns at “various global” debates.

A. joinned

B. ago

C. various

D. global

E. All correct

 

Q. 9 The “waste amount” of “information” available today has created a more knowledgeable “public”.

A. waste

B. amount

C. information

D. public

E. All correct

 

Q. 10 Some of the “steps” taken over the “late” few months have made the tax return “filing” process “simpler”.

A. steps

B. late

C. filing

D. simpler

E. All correct

 

Questions: 11 – 20

Read the following passage carefully & answer the questions given. Certain words/phrases have been given in quotes to help you locate while answering some of the questions.

A long time ago, an honest king ruled over a small kingdom named Bijanagar.  Every day, the king would listen to the complaints of his people & try to solve their problems in the best possible way. One day, a soldier came running into the king’s court. ‘I have an alarming piece of news, My Lord!’ cried out the soldier. ‘My wife told me that there is some evil in our kingdom. Crops have been “ruined” and wells have been poisoned!’ When the minister heard this, he instantly walked up to the king & said, ‘I guess some enemy is trying to create panic in our kingdom, so that our attention gets diverted to solving our internal problems. And then the enemy will take advantage of our inner disturbances & attack us,’ said the minister. You seem to be right! nodded the king. ‘What shall we do then?’ The minister said, ‘In my opinion, we should not let our people know that we suspect our enemy. It may give rise to contempt. Instead, we should distribute food & water among our people & tell them that the crops might have been damaged because of some disease. Meanwhile, I will disguise myself as an old man & try to find out the truth.’ The king agreed. That evening, the minister disguised himself as a poor old man & came out of the palace “stealthily”. The minister went to a farmer’s house & called out, ‘Food! Please give me food! The farmer came out with two pieces of bread in his hand & said, This is all that I have, O old man!’ ‘You seem to be in trouble’, said the old man. ‘Yes, someone has been very cruel. They have stamped & ruined my crops’ , said the farmer. ‘But I heard the crops were ruined because of disease,’ said the old man. ‘No, that’s not true!’ the farmer interrupted. ‘The royal guards are the culprits. They have stamped our crops & poisoned our wells’. ‘How can you be so sure?’ asked the old man. ‘I heard it from some people at the inn nearby, replied the farmer.’ The old man thanked the farmer for the food & left. He then went to the inn nearby. He saw some people gathered near a table, discussing their ruined crops & wells. Among them were two young men talking loudly. ‘Our king didn’t punish his royal guards for his wicked deed. Instead, he is trying to cover up by providing food to us!’ said one of hem. ‘Yes, our king isn’t honest any more. He has lost his goodwill!’ backed up the other. Everyone standing there nodded in agreement. After a while, the two young men prepared to leave. The minister disguised as the old man was watching them. He started following the two young men. When they came out of the inn, one of the young men said, ‘We have “provoked” these people against their king. We must continue the same way. Within a couple of days, the angry citizens will revolt against their ruler. Then we should inform our king,’ & that would be the most appropriate moment to attack’, laughed the other young man. The minister heard it all. He hid behind a tree. The two young men went inside a small hut. The minister came back to the palace & informed the king about the wicked plan of the other kingdom. Instantly the soldiers rode to the small hut where the two men are staying & captured them. the next morning, the culprits were brought out in the palace for their evil plan & were punished for their deeds.

 

Q. 11 Which of the following is most nearly the OPPOSITE in meaning to the word ‘RUINED’ as used in the story?

A. Reconstructed

B. Built

C. Succeeded

D. Nurtured

E. Accomplished

 

Q. 12 Which of the following can be an appropriate title for the story ?

A. The Ungrateful Citizens

B. The Evil in Bijanagar

C. The Good Harvest

D. Never Trust Your Minister

E. Disguise – The Only Way To Reveal The Truth

 

Q. 13 Which of the following statement is true in the context of the story ?

A. The real culprits behind the mischief were the soldiers of Bijanagar

B. The king initially hesitated to agree with the Minister’s plan

C. The minister was gifted gold coins upon identifying the culprits.

D. The farmer who gave bread to the Minister was kind & humane

E. None of the given statements is treu

 

Q. 14 Which of the following is most nearly the SAME in meaning to the word ‘PROVOKED’ as used in the story?

A. Motivated

B. Heightened

C. Instigated

D. Allowed

E. Injured

 

Q. 15 Which of the following correctly explains the meaning of the phrase, ‘lost his goodwill’ as used in the story ?

A. Misplaced his valuables

B. Lost his relatives

C. Suffered a serious injury

D. Failed to gain the love of his friends in the first place

E. Lost respect among his people

 

Q. 16 Which of the following characteristic of the Minister come across distinctly through the story ?

A. He was cowardly

B. He was sharp

C. He was greedy

D. He was old

E. He was violent

 

Q. 17 As mentioned in the story, the Minister disguised himself as an old man to :

(A) visit the kingdom without the fear of being thrashed by public

(B) participate in a local fair

(C) identify the culprit causing mischief in Bijanagar

A. only A

B. Both A and B

C. only C

D. Both B and C

E. only B

 

Q. 18 As mentioned in the story, the young were provoking the citizens of Bijanagar because :

A. they were being bribed by the minister for doing so

B. it was part of their plan to attack Bijanagar

C. they wanted to ruin the reputation of Bijanagar

D. they were notorious by nature

E. other than those given as options

 

Q. 19 Which of the following is most nearly the SAME in meaning to the word ‘STEALTHILY’ as used in the story?

A. Secretively

B. Honestly

C. Publicly

D. Shyly

E. Healthily

 

Q. 20 Which of the following is most nearly the OPPOSITE in meaning to the word ‘CONFESS’ as used in the story?

A. Hide

B. Reveal

C. Sing

D. Believed

E. Deny

 

Questions: 21 – 25

On these questions, read the sentence to find out whether there is any grammatical mistake/error in it. The error, if any, will be in one part of the sentence. Select the part with the error as your answer. If there is no error, select ‘No error’ as your answer. (Ignore the errors of punctuation, if any)

 

Q. 21 The cook quickly (1) / come inside and (2) / caught hold of (3) / the suprised crow (4) / No error (5)

A. 1

B. 2

C. 3

D. 4

E. 5

 

Q. 22 One summer afternoon (1) / the queen & her maid (2) / set out for a stroll (3) / in the palace gardens (4) / No error (5)

A. 1

B. 2

C. 3

D. 4

E. No error

 

Q. 23 All the hunters (1) / stopped in their tracks (2) /as the deer they saw (3) /was extremely beautiful. (4) / No error (5)

A. 1

B. 2

C. 3

D. 4

E. No error

 

Q. 24 As days went away, the jackal (1) / became fat & healthy because (2) / the lion was good hunter & always (3) / left a generous share for his friends (4) / No error (5)

A. 1

B. 2

C. 3

D. 4

E. No error

 

Q. 25 The merchant called out to (1) / the villagers in his high voice (2) / & it all came eagerly (3) /to see what he had with him (4) / No error (5)

A. 1

B. 2

C. 3

D. 4

E. No error

 

Questions: 26 – 30

In the following passage, there are blanks, each of which has been numbered. These numbers are printed below the passage & against each, five words are suggested, one of which fits the blank appropriately. Find out the appropriate word in each case. Once upon a time, there lived a poor orphan who was an exceptional artist & would often draw pictures of rivers, mountains & people. One day, a fairy gifted him a golden paintbrush & to his great …. 26 …… whatever he painted with the brush came to life. His fame spread & he was brought to the king’s ……..27 …….. . The king & his ministers took the boy into the fields. …..28 ……… an ocean here!’ ordered the king. The boy protested that it wasn’t right & the fields & house would be ……….29 ………. But the king ordered him to paint the ocean & a storm. The boy painted …… 30 …… & the storm was so violent that the boat in which the king & his ministers were, overturned, killing them.

 

Q. 26 Find out the appropriate word

A. doubt

B. surprise

C. miracle

D. sorrow

E. problem

 

Q. 27 Find out the appropriate word

A. favour

B. dilemma

C. court

D. justice

E. riches

 

Q. 28 Find out the appropriate word

A. sail

B. origin

C. cry

D. create

E. find

 

Q. 29 Find out the appropriate word

A. damage

B. hurt

C. form

D. destroyed

E. wiped

 

Q. 30 Find out the appropriate word

A. slowly

B. similar

C. equally

D. skilful

E. accordingly

 

Q. 31 Ruhi spends 30% of her monthly salary on rent, transportation & bills in the respective ratio of 7 : 5 : 3. If he spends Rs. 3,560/- on the transportation, how much is her monthly salary ?

A. Rs. 30,250/-

B. Rs. 40,000/-

C. Rs. 35.600/-

D. Rs.44,750/-

E. Rs. 36,000/-

 

Questions: 32 – 36

Study the table & answer the given questions.

 

Q. 32 Number of members in book club O increased by what percent from 2007 to 2010?

A. 65%

B. 64.5%

C. 58%

D. 62.5%

E. 56.5%

 

Q. 33 What is the difference between total number of members in book clubs O and P together in 2006 & that in book clubs M and N together in 2008 ?

A. 98

B. 94

C. 96

D. 104

E. 106

 

Q. 34 What is the respective ratio between total number of members in book club M in 2006 & 2010 together & that in book club Q in the same years together ?

A. 21 : 17

B. 21 : 19

C. 19 : 17

D. 23 : 19

E. 23 : 17

 

Q. 35 What is the average number of members in book clubs M, N & Q in 2007?

A. 179

B. 181

C. 177

D. 183

E. 173

 

Q. 36 If 45% of the total number of members in book clubs M, N & O together in 2009 are males, what is the total number of male members in book clubs M, N & O together in 2009

A. 181

B. 189

C. 187

D. 191

E. 197

 

Q. 37 Five years ago, the average of Simi’s age that time & Tia’s age that time was 17 years. Five years from now, the respective ratio between Simi’s age & Tia’s age that time will be 8 : 5, what is Simi’s present age?

A. 16 years

B. 11 years

C. 32 years

D. 28 years

E. 23 years

 

Q. 38 The profit earned by selling an article for Rs. 730/- is double the loss incurred when the same article is sold for Rs. 255/-, what would be the selling price of the article if it is sold at 15% profit?

A. Rs. 491/-

B. Rs. 495/-

C. Rs. 477/-

D. Rs. 483/-

E. None of these

 

Q. 39 2/5th of a number is two more than 1/3rd of another number. If the sum of the two numbers is 16. What is their product ?

A. 30

B. 120

C. 60

D. 150

E. 90

 

Q. 40 A boat can travel 12.6 km upstream in 54 min. If the speed of the water current is 1/8th of the speed of the boat in still water, how much distance (in km) the boat can travel downstream in 35 min?

A. 9

B. 10.5

C. 9.5

D. 11.5

E. 12

 

Q. 41 A circular copper wire of radius 7cm is bent to form a rectangle. If the breadth & the length of a rectangle are in the ratio of 4 : 7 respectively. What is the breadth of the rectangle ? (in cm)

A. 8 cm

B. 14 cm

C. 10 cm

D. 12 cm

E. 15 cm

 

Q. 42 5% of one number (X) is 25% more than another number (Y). If the difference between the numbers is 96 then find the value of X?

A. 90

B. 100

C. 92

D. 96

E. None of these

 

Q. 43 The sum of two positive number is 1508. If 25% of first number is equal to 40% of the second number, what is the smallest number among the two?

A. 400

B. 640

C. 580

D. 420

E. 560

 

Q. 44 Two pipes A and B can fill a tank in 16 hrs and 12 hrs respectively. The capacity of the tank is 240 litres. Both the pipes are opened simultaneously and closed after 2 hrs. How much more water (In Liters) needed to fill the tank?

A. 100

B. 170

C. 70

D. 190

E. None of these

 

Q. 45 The ratio of A’s age 3 years ago and B’s age 5 years ago is 4:5. If A is 4 years younger than B then what is the present age of B?

A. 16

B. 14

C. 11

D. 15

E. None of these

 

Q. 46 The numerator of a fraction is decreased by 25% and the denominator is increased by 250%. If the resultant fraction is 65, what is the original fraction?

A. 22/5

B. 24/5

C. 27/6

D. 28/5

E. 30/11

 

Q. 47 A dishonest vendor professes to sell fruits at the cost price but he uses a weight of 800 grams in lieu of 1 kg weight. Find his percentage gain.

A. 22%

B. 24%

C. 25%

D. 30%

E. 20%

 

Q. 48 The total of the ages of a class of 60 girls is 900 years. The average age of 20 girls is 12 years and that of the other 20 girls is 16 years. What is the average age of the remaining girls?

A. 14 years

B. 15 years

C. 16 years

D. 17 years

E. 19 years

 

Q. 49 Mr. Ankit invests 14% of his monthly income every month i.e. Rs.1,750 in shares, 8% in Insurance policies and 7% in fixed deposits. What is the total annual amount invested by him?

A. Rs. 3275

B. Rs. 3450

C. Rs. 3625

D. Rs. 3800

E. none of these

 

Q. 50 0.5% of 674 of 0.8% of 225 = ?

A. 7.066

B. 9.12

C. 6.066

D. 5.17

E. None of these

 

Q. 51 Choose correct option in given series.

1, 4, 27, 16, 125, 36, ?

A. 512

B. 729

C. 343

D. 216

E. 1331

 

Q. 52 Choose correct option in given series.

0, 6, 24, 60, ? , 210

A. 120

B. 125

C. 211

D. 59

E. 338

 

Q. 53 Choose correct option in given series.

3645, 1215, 405, 135, ? , 15, 5

A. 75

B. 45

C. 65

D. 55

E. 35

 

Q. 54 Choose correct option in given series.

15, 30, ? , 40, 8, 48

A. 10

B. 20

C. 18

D. 12

E. 13

 

Q. 55 Choose correct option in given series.

5, 13, 58, 357, 2868, ?

A. 25823

B. 28695

C. 29548

D. 28545

E. 27695

 

Questions: 56 – 60

What should come in place of question mark (?) in the following questions?

 

Q. 56 36%of 170+? %of 592 = 150

A. 14.6

B. 17.8

C. 16.4

D. 15

E. None of these

 

Q. 57 (12.11)2 + (?)2 = 732.2921

A. 20.2

B. 24.2

C. 23.1

D. 19.2

E. None of these

 

Q. 58 288 ÷ ?x 57 = 2052

A. 8

B. 7

C. 6

D. 9

E. None of these

 

Q. 59 (1125—274—323)÷(875—654— 155) =?

A. 9

B. 6

C. 7

D. 8

E. None of these

 

Q. 60 (42)2 ÷ 6.3 x 26 =?

A. 7182

B. 7269

C. 7260

D. 7240

E. None of these

 

Q. 61 How many different words can be formed with the letters of the word UNIVERSITY so that all the vowels are together?

A. 60840

B. 60480

C. 60460

D. 40680

E. None of these

 

Q. 62 There are 5 positive observations. Average of first 3 observations is 8 & that of the last 3 observations is 6. If the average of all five observation is 6, what is the third observation ?

A. 10

B. 8

C. 12

D. 14

E. 6

 

Q. 63 Dan invested certain sum in scheme A, which offers simple interest at 8% p.a. for 4 years. He also invested Rs. 4,000/- in scheme B, which offers compound interest (compounded annually), at 10% p.a. for 2 years. If the interest earned from scheme A is 4/5th of the interest earned from scheme B, what is the sum invested in scheme A ?

A. Rs. 3,400/-

B. Rs. 2,100/-

C. Rs. 3,250/-

D. Rs. 3,650/-

E. None of these

 

Q. 64 The perimeter of an equilateral triangle is 40 m less than than the perimeter of a square. If the respective ratio between the side of a triangle & the side of a square is 8 : 11, what is the area of the square? (in m^2)

A. 441

B. 324

C. 289

D. 576

E. 484

 

Q. 65 A & B can independently finish a piece of work in 30 days & 40 days respectively. They started working together & after few days B left. After that A could finish the work in 16 days. After how many days of working together B left ?

A. 10

B. 6

C. 12

D. 4

E. 8

 

Q. 66 All the letters of the word JUNKYARD are arranged in alphabetical order from left to right. Then all the vowels are replaced with next alphabet (as per the english alphabetical order), then which of the following will be the second letter from the right end ?

A. V

B. D

C. K

D. N

E. R

 

Questions: 67 – 68

Study the following information & answer the given questions.

M is the father of V.

V is the sister of X.

X is married to R.

R is the daughter of P.

P is the wife of H.

 

Q. 67 If N is the only child of V, then how is N related to M ?

A. Grandfather

B. Daughter

C. Son

D. Cannot be determined

E. Brother

 

Q. 68 How is X related to H ?

A. Daughter-in-law

B. Nephew

C. Daughter

D. Son-in-law

E. Son

 

Questions: 69 – 73

Study the following information & answer the given questions.

J, K, L, M, N, O, P and Q are sitting around a circular table facing the centre but not necessarily in the same order.

Only 3 people sit between Q and P. N sits third to the right of O. O is neither an immediate neighbour of Q nor P.

J is an immediate neighbour of P. Only 3 people sit between J and K.

Only 1 people sits between K and M. M is not an immediate neighbour of Q.

 

Q. 69 Who among the following sits second to the left of O ?

A. K

B. L

C. M

D. P

E. Q

 

Q. 70 Who among the following represent the immediate neighbour of Q ?

A. M, K

B. K, N

C. L, K

D. J, N

E. J, L

 

Q. 71 Who among the following is true regarding L, as per the given arrangement ?

A. Only 3 people sit between O and L

B. None of the given statement is true

C. N sits second to the left of L

D. P is one of the immediate neighbours of L

E. Only one person sits between L and K

 

Q. 72 Four of the following five are alike in a certain way based on their position in the given arrangement & so form a group. Which is the one that does not belong to that group ?

A. LKN

B. POM

C. MKO

D. NJL

E. KNQ

 

Q. 73 How many people are seated between O and J, when counted from the left of J ?

A. None

B. One

C. Two

D. Three

E. Four

 

Q. 74 In a straight line of 12 people (facing North), P sits fifth from the left end of the line. Only 3 people sit between J and P. S sits second to the right of J. O sits second to the right of S. How many people sit between O and P ?

A. None

B. Cannot be determined

C. Three

D. One

E. Two

 

Q. 75 If all the numbers in 42731658 are arranged in ascending order from left to right, the position(s) of how many number(s) will remain unchanged ?

A. Three

B. More than three

C. None

D. Two

E. One

 

Questions: 76 – 80

These questions are based on five three digit numbers given below.

415 764 327 542 256

 

Q. 76 What will be the resultant if second digit of the lowest number & third digit of the highest number are multiplied?

A. 30

B. 14

C. 8

D. 20

E. 36

 

Q. 77 If ‘1’ is added to the first digit of every odd number & ‘2’ is subtracted from second digit of every even number, in how many numbers will a digit appears twice ?

A. Two

B. Three

C. Four

D. None

E. One

 

Q. 78 The positions of the first & the second digits of each of the numbers are interchanged. What will be the resultant if third digit of highest number thus formed is divided by the second digit of the lowest number thus formed ?

A. 3

B. 4

C. 2.5

D. 1.5

E. 1

 

Q. 79 If in each number all the digits are arranged in ascending order from left to right within the number, how many numbers thus formed will be odd numbers ?

A. None

B. Two

C. One

D. Four

E. Three

 

Q. 80 If all the numbers are arranged in ascending order from left to right, which of the following will be the sum of all the three digits of the number which is third from left ?

A. 12

B. 11

C. 10

D. 13

E. 17

 

Questions: 81 – 85

In these questions, relationship between different element is shown in the statements. The statements are followed by conclusions. Study the conclusion based on the given statements & select the appropriate answer.

 

Q. 81 Statement : F < R < L ≤ S > O

Conclusions : I. F < S II. O > R

A. If either conclusion I or II is true.

B. If only conclusion I is true.

C. If both conclusions I & II are true.

D. If neither conclusion I nor II is true.

E. If only conclusion II is true.

 

Q. 82 Statement : U ≤ C = n < Q ≥ J

Conclusions : I. Q > U II. C < J

A. If either conclusion I or II is true.

B. If only conclusion I is true.

C. If both conclusions I & II are true.

D. If neither conclusion I nor II is true.

E. If only conclusion II is true.

 

Q. 83 Statement : G ≥ R = O ≥ W

Conclusions : I. G > W II. W = G

A. If either conclusion I or II is true.

B. If only conclusion I is true.

C. If both conclusions I & II are true.

D. If neither conclusion I nor II is true.

E. If only conclusion II is true.

 

Q. 84 Statement : K > E ≥ R = A; E < B

Conclusions : I. K = A II. A< B

A. If either conclusion I or II is true.

B. If only conclusion I is true.

C. If both conclusions I & II are true.

D. If neither conclusion I nor II is true.

E. If only conclusion II is true.

 

Q. 85 Statement : D = O < L ≤ P > H

Conclusions : I. P < D II. O > H

A. If either conclusion I or II is true.

B. If only conclusion I is true.

C. If both conclusions I & II are true.

D. If neither conclusion I nor II is true.

E. If only conclusion II is true.

 

Questions: 86 – 90

In these questions, a group of numbers/symbols followed by five combinations of inter codes is given. You have to find out which of the combinations correctly represents the group of numbers/symbols based on the given coding system & the conditions & mark that combination as your answer.

Conditions :

(i) If the first & the third element are symbols then their codes are to be interchanged.

(ii) If an odd number is immediately followed as well as immediately preceded by a symbol then that odd number is to be coded as ‘X’.

(iii) If the last element is an even number then the first element is to be coded as the code of that even number.

(iv) If the second element is symbol then the code of that symbol is to be interchanged with the code of the first element.

(Note : All the elements have to be counted from left to right to fulfil the condition.)

 

Q. 86 @ ^ 2 * 43

A. DPLMUS

B. MPXDUS

C. PDLMUS

D. MPLDUS

E. PDXMUS

 

Q. 87 64 # % 9 &

A. FUYJXZ

B. YUFZQJ

C. FUYZXJ

D. JUYFQZ

E. YUFXQJ

 

Q. 88 * 2 # @ 87

A. LYMDUA

B. YLMXUA

C. YLMDUA

D. MXYDUA

E. MLYDUA

 

Q. 89 & 4 % # 27

A. UJZYLA

B. JKZYLA

C. ZUJYUA

D. ZUJYLA

E. JUZYLA

 

Q. 90 + 95 # 68

A. YQCRFU

B. YQCYFU

C. UQCRXU

D. QRCYFU

E. UQCYFU

 

Questions: 91 – 95

Study the following information to answer the given questions.

J, K, L, M, P, Q and R are seated in a straight line facing North, with equal distance between each other but not necessarily in the same order.

P sits third to the left of K. Q sits second to the right of K. L sits third to the right of R. R does not sit at any of the extreme ends of the line.

Only one person sits between R and J.

 

Q. 91 What is the position of J with respect to P ?

A. Immediate left

B. Third to the left

C. Fourth to the left

D. Fifth to the right

E. Second to the right

 

Q. 92 How many persons are seated between R and Q ?

A. Two

B. Three

C. None

D. Four

E. One

 

Q. 93 Who amongst the following sits to the immediate right of L ?

A. P

B. Q

C. K

D. J

E. M

 

Q. 94 Who amongst the following sits exactly in the middle of the line ?

A. M

B. R

C. P

D. Q

E. J

 

Q. 95 Which of the following represents persons seated at two extreme ends of the line?

A. J, Q

B. K, Q

C. L, P

D. K, L

E. P, K

 

Q. 96 In a certain code language, DEALT is coded as FDCKV and HOPES is coded as JNRDU. In the same code language, how will CHALK be coded as ?

A. EFGJM

B. EGCKM

C. BFCKU

D. EGCJL

E. BFCJM

 

Questions: 97 – 98

Study the following information carefully to answer the given questions.

Five people A, B, C, D and E have different numbers of flowers in their hands. B has more flowers than only one person. A has more flowers than C but less than D. C does not have the least number of flowers. The one who has the most number of flowers has 17 flowers. The one who has the third least number of flowers has 11 flowers.

 

Q. 97 If C has 4 more flowers than B, then how many flowers does B have ?

A. 7

B. 15

C. 18

D. 13

E. 21

 

Q. 98 Who amongst the following has the second most number of flowers ?

A. D

B. A

C. B

D. E

E. C

 

Q. 99 In a certain code language, ‘very good dancers’ is coded as ‘325’. Similarly, ‘good bike stunts’ is coded as ‘734’ and ‘dancers doing stunts’ is coded as ‘792’. What will be the code for ‘doing’ in the given code language ?

A. Either ‘2’ or ‘7’

B. 2

C. 7

D. 9

E. Either ‘7’ or ‘9’

 

Q. 100 How many such pairs of letters are there in the word FASTER, each of which has as many letters between them in the word (in both forward & backward directions) as they have between them in the english alphabetical series ?

A. More than three

B. None

C. Two

D. One

E. Three

 

 

Answer Sheet
Question 1 2 3 4 5 6 7 8 9 10
Answer E D A B D B C A A B
Question 11 12 13 14 15 16 17 18 19 20
Answer D B D C E B C B A E
Question 21 22 23 24 25 26 27 28 29 30
Answer B E A A B B C D D E
Question 31 32 33 34 35 36 37 38 39 40
Answer C D C A C B D D C B
Question 41 42 43 44 45 46 47 48 49 50
Answer A B C B D D C D D D
Question 51 52 53 54 55 56 57 58 59 60
Answer C A B A B D B A E E
Question 61 62 63 64 65 66 67 68 69 70
Answer B C B E E A D D A B
Question 71 72 73 74 75 76 77 78 79 80
Answer A A E A D D B E D C
Question 81 82 83 84 85 86 87 88 89 90
Answer B B A E D C C C D E
Question 91 92 93 94 95 96 97 98 99 100
Answer E B B A A B A B D C

IBPS Clerk 2015 Prelims Previous Year Paper

IBPS Clerk 2015 Prelims Paper 1

Section

Questions

Marks

English

30 Questions (1 – 30)

30

Logical Reasoning

30 Questions

30

Quantitative Aptitude

15 Questions

15

Data Interpretation

5 Questions (96 – 100)

5

 

Questions: 1 – 5

Rearrange the following five sentences (A), (B),(C),(D) and (E) in a proper sequence to form a meaningful paragraphs, then answer the given questions.

(A) The big orange truck was playing music and had bells that chimed repeatedly.

(B) When Lisa approached the truck, the man inside it said, “Here you go, young lady, enjoy your ice-cream sandwich/”

(C) Lisa was amazed and said “What a brilliant idea this is; don’t wait for your customers to come to you; go out and find them!”

(D) Several people attracted by the bells were walking up to it and speaking to a man who stood inside it.

(E) One day, Lisa walked to the front of her house and was surprised to see a big orange truck on the street.

 

Q. 1 Which of the following should be the SECOND sentence after the arrangement?

A. A

B. B

C. C

D. D

E. E

 

Q. 2 Which of the following should be the FIRST sentence after the arrangement?

A. A

B. B

C. E

D. D

E. C

 

Q. 3 Which of the following should be the FOURTH sentence after the arrangement?

A. A

B. B

C. C

D. E

E. D

 

Q. 4 Which of the following should be the THIRD sentence after the arrangement?

A. A

B. B

C. C

D. D

E. E

 

Q. 5 Which of the following should be the LAST (FIFTH) sentence after the arrangement?

A. A

B. E

C. C

D. D

E. B

 

Questions: 6 – 15

A long time ago, there lived a gardener. He was hardworking and honest. He had planted different types of flowers and vegetables in his garden. He had a younger brother who was foolish and lazy. He would never help the gardener in his work. Instead, he would say, ‘There is no need to work so hard. Brother! Nature will take care of itself.’ Once is so happened that the gardener had to go to a nearby town for a week. ‘Please water the plants and trees in my absence!’ said the gardener to his younger brother. ‘Some plants need to be watered daily and some others only on alternate days.’ the gardener explained. But his words fell on deaf ears. The gardener left for the town. Now the lazy brother thought, ‘Why should I bother about these plants and trees? My brother is away and he wouldn’t know whether I am watering his garden or not. “Then, he went to sleep as usual. Two days had passed. Now the small plants had started dying up. The lazy fellow glanced at them and laughed. ‘Hahaha! Poor dumb plants! They cannot even complain to my brother that I am not watering them!’ After a week when the gardener returned, he was shocked to see the miserable condition of his garden. Most of the plants had died, the climbers had dried and the trees looked diseased. ‘So it seems you didn’t water the plants,’ he yelled at his lazy brother. ‘I did. But the plants were of bad quality, so they died!, said the lazy brother. The gardener knew that it was no use talking to his brother. So he cleaned his garden and nursed the diseased trees. He then went to the market and brought some seeds. When his brother saw him preparing to sow the seeds, he came near him. ‘And that’s it. Brother! Don’t I tell you that nature takes care of itself? See those seeds! You will sow them and once again your garden will be ready as green as ever.’ ‘You are right’,’ said the gardener. ‘But have you ever thought about where these seeds came from?, ‘From the market, of course!’ replied his brother. ‘And how did they reach the market?’ the gardener asked again. This time his brother had no reply, ‘Uuuuhm!’ he kept searching for words. ‘I’ll tell you! said the gardener. ‘The seeds also come from plants and trees. If we don’t take proper care of the plants, the seeds too will die. And a day will come when there will be no seeds to sow. How will you grow your plants then? And how will you get vegetables and fruits? Just think! Won’t you die of hunger? ‘Oh, I never thought that way!’ the gardener’s brother shrieked. ‘It’s true that nature takes care of itself. And it cares for us too. That’s why it has given us seeds. A small seed contains the biggest secret of nature. All that we need to do is to unfold that secret. It has so much power in it. It can make a beautiful tree, with flowers and fruits and more seeds,’ explained the gardener. Now his brother realised his mistake. From that day on, he was not lazy anymore. He started helping his brother in nurturing the plants and trees. He was now well aware of the secret of nature.

 

Q. 6 Which of the following statement is true in the context of the story?

A. The gardener’s brother was two years older than him.

B. Before leaving for the town, the gardener knew this brother would fail to complete the task assigned to him.

C. The gardener’s brother was jealous of him since childhood.

D. The gardener did not explain how to take care of his plants; because of this his plants died.

E. None of the given statements is true

 

Q. 7 Choose the word which is most nearly the OPPOSITE in meaning of the word ‘AWARE’ as used in the passage?

A. absent

B. short

C. ignorant

D. knowing

E. agree

 

Q. 8 Which of the following correctly explains the meaning of the phrase ‘fell on deaf ears’ as used in the story?

A. Were heard and accepted

B. Were ignored

C. Fell head first

D. Were deafeningly loud

E. Were too soft

 

Q. 9 As mentioned in the story, the gardener had to go to a town nearby because

A. he wanted to get a different variety of goods.

B. he wished to visit one of his old friends.

C. his parents were unwell and he went to visit them.

D. he wanted to build a house for his brother.

E. not clearly mentioned in the story.

 

Q. 10 Which of the following is most nearly the OPPOSITE in meaning of the word ‘NURTURING’ as used in the passage?

A. starving

B. believing

C. supplying

D. dieting

E. watering

 

Q. 11 Which of the following is most nearly the SAME in meaning as the word ‘MISERABLE’ as used in the passage?

A. bankrupt

B. cherished

C. poor

D. denial

E. scanty

 

Q. 12 As mentioned in the story, the gardener’s brother discouraged his brother from taking care of his garden because

(A) he did not want his brother to struggle so much.

(B) he was planning to hire a servant for his brother.

(C) for him, it was a futile effort

A. Only (A)

B. Both (A) and (B)

C. Only (B)

D. Only (C)

E. Both (B) and (C)

 

Q. 13 Which of the following characteristics of the gardener’s brother comes across distinctly at the end of story?

A. He started respecting his parents.

B. He became violent.

C. His hatred towards his brother amplified

D. He became depressed

E. He became sensitive towards nature.

 

Q. 14 Which of the following is most nearly the SAME in meaning as the word ‘BOT HER’ as used in the passage?

A. worry

B. content

C. apprehend

D. doubt

E. kin

 

Q. 15 Which of the following can be an appropriate title for the story?

A. Too Late to Repair the Damage

B. The Jealous Brother

C. The Idle Gardener

D. The Secret of Nature

E. The Fate of Disappearing Plants

 

Questions: 16 – 20

Read this sentence to find out if there is any grammatical mistake/error in it. The error, if any, will be in one part of the sentence. Mark the part with the error as your answer. If there is no error mark, mark ‘No error’ as your answer.

 

Q. 16 There was / once a gardener / who took care / of the king’s garden.

A. There was

B. once a gardener

C. who took care

D. of the king’s garden

E. No error

 

Q. 17 By and by, the fox / comes to a stream / that ran through the forest, / and quenched his thirst.

A. By and by, the fox

B. comes to a stream

C. that ran through the forest,

D. and quenched his thirst

E. No error

 

Q. 18 The giant entered / the forest at night / when all the / animals were asleep.

A. The giant entered

B. the forest at night

C. when all the

D. animals were asleep

E. No error

 

Q. 19 The ant realise that / a pigeon sitting on the tree / had dropped a leaf / into the water to save him.

A. The ant realise that

B. a pigeon sitting on the tree

C. had dropped a leaf

D. into the water to save him

E. No error

 

Q. 20 While trying to pick up / the few fallen ones, / the monkey dropped almost / all the fruits encased his hands.

A. While trying to pick up

B. the few fallen ones,

C. the monkey dropped almost

D. all the fruits encased his hands

E. No error

 

Questions: 21 – 25

In this question, a sentence with four words in bold type is given. One of these four words given in bold may be either wrongly spelt or inappropriate in the context of the sentence. Find out the word which is wrongly spelt or inappropriate. That word is your answer

 

Q. 21 Leaders and managers must communicate well with employees, customers, investors and society as a whole.

A. mangers

B. communicate

C. employees

D. whole

E. All correct

 

Q. 22 The government will push through economic reforms once the knew financial year begins.

A. push

B. knew

C. financial

D. begins

E. All correct

 

Q. 23 Committees have been asked to dispose of all grievances petitions within the next fortnight.

A. dispose

B. petitions

C. within

D. fortnight

E. All correct

 

Q. 24 Generally gift cards do not allow a cash refund and have a validity period.

A. Generally

B. allow

C. validity

D. period

E. All correct

 

Q. 25 Printing money to boost the economy is a threat to financial stability.

A. Printing

B. threat

C. financial

D. stablety

E. All correct

 

Questions: 26 – 30

In the given passage, there are blanks each of which has been numbered. Against each, five words are suggested, one of which fits the blank appropriately. Find out the appropriate word in each case.

Once upon a time there lived a spider named Anansi who was gifted a pot of wisdom to share among all the creatures of the world. But Anansi was unwilling to share it with anybody and decided to (26) the pot at the top of a tall tree . With the pot at his waist in front, it was (27) for Anansi to climb. Seeing his father’s awkward ascent, Anansi’s son was (28) and advised him ‘Father, why don’t you carry the pot on your back (29)? Anansi did so and found it easy to scale the tree. He (30) that though he had a pot of wisdom he lacked common sense. Angry at himself he threw the pot down, where it broke into pieces and wisdom, scattered among all the creatures.

 

Q. 26 What word should be in (26)

A. disappear

B. vanish

C. hide

D. observe

E. unearth

 

Q. 27 What word should be in (27)

A. tired

B. difficult

C. challenge

D. risk

E. safely

 

Q. 28 What word should be in (28)

A. laughing

B. worrying

C. anxiously

D. busy

E. amused

 

Q. 29 What word should be in (29)

A. lied

B. alternative

C. subsequent

D. instead

E. perhaps

 

Q. 30 What word should be in (30)

A. realised

B. achieved

C. recognise

D. aware

E. understand

 

Questions: 31 – 35

In this question, a group of numbers/symbols followed by five combinations of letter codes is given. You have to find out which of the combinations correctly represents the group of the numbers/symbols based on the given coding system and the conditions and mark the combination as your answer.

Conditions :-

I. If the first and the last elements are symbols then their codes are to be interchanged.

II. I f a symbol is immediately followed as well as immediately preceded by a number then that symbol is to be coded as ‘1’

III. If the last element is an odd number then the second element is to be coded as the code of the odd number.

IV. If the third element is an even number then the code of that even number is to be interchanged with the code of the first element.

(Please note: All the elements have to be counted from left to right to fulfill the conditions.)

 

Q. 31 #@3+42

A. ZDX1HE

B. EOKUHZ

C. ZKE1DH

D. EDX1HZ

E. EDUXHZ

 

Q. 32 8∧@587

A. LMDA1M

B. LTMAKM

C. DTLAKM

D. LMKDAM

E. DTUXAM

 

Q. 33 @32

A. EXDPZL

B. EZDPXL

C. XEDZ1L

D. XLEZPL

E. 2LEXPL

 

Q. 34 @97%∧δ

A. LYDPTK

B. DY1PTK

C. KYMPTD

D. DYL1TK

E. LYDPTG

 

Q. 35 $452#%

A. GH1EZB

B. GHA1EZ

C. BHZEAG

D. BHAEZG

E. AHGEZB

 

Q. 36 In a certain code language, ‘avoid going out’ is coded as ‘319’. Similarly, ‘going for party’ is coded as ‘612’ and ‘avoid for party’ is coded as ‘362’. What will be the code for the ‘party’ in the given code language?

A. 3

B. 1

C. 9

D. Either ‘3’ or ‘1’

E. Either ‘2’ or ‘6’

 

Questions: 37 – 41

S, T, U, V, X, Y, and Z are sitting around a circular table facing the centre but not necessarily in the same order. S sits third to the right of T. Only three people sit between U and X. U is neither an immediate neighbour of S nor of T. Only three people sit between T and W. Z sits on the immediate right of W. V sits third to the left of W.

 

Q. 37 How many people are seated between S and Z, when counted from the right of S?

A. None

B. Two

C. Four

D. Three

E. One

 

Q. 38 Who among the following represent the immediate neighbours of S?

A. X, Y

B. X, Z

C. W, Y

D. W, X

E. V, Y

 

Q. 39 Who among the following sits second to the left of Y?

A. U

B. X

C. W

D. Z

E. S

 

Q. 40 Four of the following five are alike in a certain way based on their positions in the given arrangement and so form a group. Which is the one that does not belong to the group?

A. UYV

B. TVY

C. SWU

D. XSZ

E. VXW

 

Q. 41 Which amongst the following is true regarding V, as per the given arrangement?

A. None of the given statements is true.

B. Only three people sit between V and S.

C. U sits second to the right of V

D. V sits second to the right of U

E. Only two people sit between V and Z.

 

Questions: 42 – 43

Five cars P, Q, R, S and T each has a different mileage. P has more mileage than only one car. Both Q and T have more mileage than P but less than S. Q has more mileage than R but less than T. The car which has the highest mileage runs for 22 km/litre. The car which has the third lowest mileage runs for 19 km/litre.

 

Q. 42 Which amongst the following cars has the highest mileage?

A. S

B. Q

C. Cannot be determined

D. T

E. R

 

Q. 43 If the difference between the mileage of R and that of Q is of 4 km/litre, then what is the mileage of R?

A. 23 km/litre

B. 21 km/litre

C. 18 km/litre

D. 15 km/litre

E. 12 km/litre

 

Q. 44 In each question, a relationship between different element is show in the statement(s). The statements are followed by two conclusions. Study the conclusions based on the given statements(s) and select the appropriate answer.

Statement : Q ≤ U < 1 ≤ C = K

Conclusions : I. U > K II. Q < C

A. Both conclusion I and II are true

B. Only conclusion I is true

C. Either conclusion I or II is true

D. Neither conclusion I nor II is true

E. Only conclusion II is true

 

Q. 45 In each question, a relationship between different element is show in the statement(s). The statements are followed by two conclusions. Study the conclusions based on the given statements(s) and select the appropriate answer.

Statements : T = A < X ≤ S ; E < X

Conclusions : I. T < S II. T > E

A. Only conclusion II is true

B. Either conclusion I or II is true

C. Neither conclusion I nor II is true

D. Only conclusion I is true

E. Both conclusion I and II are true

 

Q. 46 In each question, a relationship between different element is show in the statement(s). The statements are followed by two conclusions. Study the conclusions based on the given statements(s) and select the appropriate answer.

Statements : H ≥ Y ≥ P = E > R

Conclusions : I. E < H II. Y > R

A. Both conclusion I and II are true

B. Only conclusion II is true

C. Neither conclusion I nor II is true

D. Either conclusion I or II is true

E. Only conclusion I is true

 

Q. 47 In each question, a relationship between different element is show in the statement(s). The statements are followed by two conclusions. Study the conclusions based on the given statements(s) and select the appropriate answer.

Statements : J = U ≤ N ≤ E

Conclusions : I. J < E II. E = J

A. Both conclusion I and II are true

B. Only conclusion I is true

C. Only conclusion II is true

D. Either conclusion I or II is true

E. Neither conclusion I nor II is true

 

Q. 48 In each question, a relationship between different element is show in the statement(s). The statements are followed by two conclusions. Study the conclusions based on the given statements(s) and select the appropriate answer.

Statements : D < O > N > K < Y

Conclusions : I. D > K II. Y < O

A. Either conclusion I or II is true

B. Only conclusion I is true

C. Both conclusion I and II are true

D. Neither conclusion I nor II is true

E. Only conclusion II is true

 

Questions: 49 – 53

This question is based on the five three – digit numbers given below.

528 247 724 285 856

 

Q. 49 If ‘2’ is added to the first digit of every odd number and ‘3’ is subtracted from the third digit of every even number, then in how many numbers will a digit appear twice?

A. Three

B. One

C. Two

D. None

E. Four

 

Q. 50 If all the numbers are arranged in ascending order from left to right, which of the following will be sum of all the three digits of the number which is third from the right?

A. 13

B. 16

C. 15

D. 14

E. 19

 

Q. 51 What will be the resultant if the third digit of the lowest number and the second digit of the highest number are multiplied?

A. 42

B. 24

C. 10

D. 35

E. 20

 

Q. 52 The positions of the first and the third digit of each of the numbers are interchanged. What will be resultant if the first digit of the lowest number thus formed is divided by the second digit of the highest number thus formed?

A. 1

B. 2

C. 1.5

D. 2.5

E. 3

 

Q. 53 If in each number all the digits are arranged in descending order from left to right within the number, how many numbers thus formed will be odd number?

A. None

B. Three

C. Four

D. One

E. Two

 

Questions: 54 – 55

Study the following information carefully and answer the given questions. P is the father of D. D is the only son of T. T is the daughter of J. T is the mother of G. G is the sister of V.

 

Q. 54 If J is married to B, then how is b related to G?

A. Daughter – in – law

B. Son – in – law

C. Father

D. Mother

E. Cannot be determined

 

Q. 55 How is V related to P?

A. Daughter

B. Father

C. Mother

D. Cannot be determined

E. Son

 

Q. 56 How many such pairs of letters are there in the word TIMELY each of which has  as many letters between them in the word ( in both forward and backward directions) as they have between them in the English alphabetical series?

A. Two

B. None

C. Three

D. One

E. More than three

 

Q. 57 If all the numbers in 86312749 are arranged in ascending order from left to right, the position of how many numbers will remain unchanged?

A. Three

B. One

C. Two

D. More than three

E. None

 

Questions: 58 – 62

G, H, I, J, S, T, and U are seated in a straight line facing north, with equal distance between each other, but not necessarily in the same order.

I sits second to the right of T. U sits third to the right of I. H sits third to the left of S. S does not sit at any of the extreme ends of the line. Only one person sits between S and G.

 

Q. 58 Who among the following sits exactly in the middle of the line?

A. G

B. J

C. U

D. I

E. S

 

Q. 59 How many persons are seated between I and S?

A. One

B. Two

C. Four

D. None

E. Three

 

Q. 60 Which of the following represents persons seated at the two extreme ends of the line?

A. G, U

B. T, G

C. G, J

D. I, J

E. H, I

 

Q. 61 What is the position of U with respect to G?

A. Second to the right

B. Immediate left

C. Fourth to the left

D. Fifth to the right

E. Third to the left

 

Q. 62 Who among the following sits on the immediate left of H?

A. J

B. G

C. I

D. U

E. T

 

Q. 63 In a certain code language, BLUNT is coded as AKSOU and COINS is coded as BNGOT. In the same code language, how will TRUST be coded as ?

A. SSTUV

B. SQTRS

C. RQSRS

D. SQSTU

E. RSSQN

 

Q. 64 All the letters of the word HALFTIME are arranged in alphabetical order from left to right. Then all the vowels are replaced with the next alphabet (as per the English alphabetical order). Which of the following will be the second letter from the left end?

A. H

B. F

C. M

D. L

E. E

 

Q. 65 There are six persons seated in a row (facing north). C sits sixth from the right end of the line. Only three persons sit between C and M. B sits second to the right of X. Only two persons sit between N and K.

How many persons sit between B and C?

A. One

B. Three

C. Cannot be determined

D. Four

E. Two

 

Q. 66 Six years from now, the average of Monu’s age that time and Ninu’s age that time will be 29 years. Five years ago, if the ratio of Monu’s age to Nine’s age that time was 11 : 7, what is Ninu’s present age?

A. 17 years

B. 33 years

C. 27 years

D. 19 years

E. 22 years

 

Q. 67 Rahim spends 60% of his monthly salary on rent, EMI and miscellaneous expenses in the ratio of 2 : 1 : 3. If he spends a total of Rs.16050 on rent and EMI together, how much is his monthly salary?

A. Rs. 50300

B. Rs.49600

C. Rs.46750

D. Rs.53500

E. Rs.54500

 

Q. 68 There are seven positive observations. The average of the first four observations is 11 and that of the last four observations is 8. If the average of these seven observations is 9, what is the fourth observation?

A. 7

B. 9

C. 8

D. 11

E. 13

 

Q. 69 Prem invested a certain sum in Scheme A, which offers simple interest at the rate of 8% per year for 4 years. He also invested Rs. 2000 in Scheme B, which offers compound interest (compounded annually) at 20% pa for 2 years. If the interest earned from scheme A is 1 7/11 of the interest earned from scheme B, what is the sum invested in scheme A?

A. Rs. 4000

B. Rs. 3000

C. Rs. 4500

D. Rs. 3600

E. None of these

 

Q. 70 Two trains started running from the same point at the same time in opposite directions (one towards North and the other towards South). The speed of the two trains is 22 m/s and 8m/s respectively. How much time will they take to be 378 km apart?

A. 3h 30 minutes

B. 3h 20 minutes

C. 4h 30 minutes

D. 4h 52 minutes

E. None of these

 

Q. 71 2/7 of a number is two less than 1/2 of another number.

If the sum of two numbers is 15, what is their product?

A. 72

B. 64

C. 54

D. 63

E. 56

 

Q. 72 What will come in place of question mark (?) in he given question?

9² ✕ 7² ÷ √441 – 64 = 5ˀ

A. 4

B. 2

C. 5

D. 6

E. 3

 

Q. 73 What will come in place of question mark (?) in the given question?

(⅘ + 1⅞ + ⅝) of ? = 759

A. 290

B. 210

C. 270

D. 230

E. 250

 

Q. 74 What will come in place of question mark (?) in the given question?

√2601 + √169 = 8¹²⁻ˀ

A. 10

B. 9

C. 8

D. 7

E. 6

 

Q. 75 What will come in place of question mark (?) in the given question?

(125.5 + 242.75 + ?) ✕ 6/7 = 480

A. 191.75

B. 172.75

C. 219.75

D. 189.25

E. 211.25

 

Q. 76 What will come in place of question mark (?) in the given question?

√(121 ✕ 5 + 133 – 657) = ?

A. 4

B. 6

C. 3

D. 5

E. 9

 

Q. 77 What will come in place of question mark (?) in the given question?

45% of 360 + 288 = ?% of 750

A. 65

B. 60

C. 70

D. 75

E. 65

 

Q. 78 What will come in place of question mark (?) in the given question?

? + (8⅐ ✕ 6⁵/₁₉) = 5³

A. 56

B. 66

C. 64

D. 91

E. 74

 

Q. 79 What will come in place of question mark (?) in the given question?

35% of 580 + 70% of ? = 441

A. 380

B. 340

C. 360

D. 320

E. 280

 

Q. 80 What will come in place of question mark (?) in the given question?

40 ÷ 4.8 ✕ 78 + ? = 1480

A. 670

B. 930

C. 650

D. 870

E. 830

 

Q. 81 What will come in place of question mark (?) in the given question?

(0.6 x 450) /5 = 2 x 3ⁱ

Find the value of i?

A. 3

B. -3

C. -2

D. -1

E. 2

 

Q. 82 The selling price of 16 chairs is equal to the selling price of 6 tables. If the total selling price of 5 chairs and 3 tables together is Rs. 780, what is the total selling price of 2 chairs and 5 tables together?

A. Rs. 920

B. Rs. 960

C. Rs. 980

D. Rs. 1060

E. Rs. 860

 

Q. 83 The sum of two positive numbers is Rs. 630. If 75% of the first number is equal to 60% of the second number, what is the larger number among the two?

A. 280

B. 340

C. 350

D. 420

E. 270

 

Q. 84 A boat can travel 10.2 km upstream in 51 minutes. If the speed of the current is 1/5 of the speed of the boat in still water, how much distance (in km) can the boat travel downstream in 48 minutes?

A. 14.8

B. 15.6

C. 15.2

D. 17.4

E. 14.4

 

Q. 85 A started a business by investing Rs. 4200 and after 2 months B joined by investing Rs. 3000. At the end of 4 months from the start of the business, C joined with an investment of Rs. 6000. At the end of 10 months from the start of business, A added an additional amount of Rs. 1800. If A’s share in the annual profit was Rs. 1620, what was the total annual profit ?

A. Rs. 3960

B. Rs. 3080

C. Rs. 4070

D. Rs. 3320

E. Rs. 4180

 

Q. 86 (A) and (B) can independently finish a piece of work in 36 days and 45 days respectively. They started working together and after a few days (A) left. After that (B) could finish the remaining work in 36 days. After how many days of working did (A) leave?

A. 6 days

B. 5 days

C. 3 days

D. 4 days

E. 8 days

 

Q. 87 The perimeter of an equilateral is 21m more than the perimeter of a square. If the ratio of the side of the triangle to the side of the square is 9 : 5, what is the area of the square? (in m²)

A. 25

B. 225

C. 625

D. 144

E. 81

 

Q. 88 A circular copper wire of radius 35 cm is bent to form a rectangle. If the length of the rectangle is more than the breadth of the rectangle by 27 cm, what is the length of the rectangle ? (in cm)

A. 77

B. 64

C. 76

D. 62

E. 68

 

Q. 89 In what ratio of rice of variety A worth Rs. 36 must be mixed with rice of variety B worth Rs. 48, so that the new mixture (of both the varieties A & B) is worth Rs.45?

A. 1 : 3

B. 3 : 4

C. 4 : 5

D. 1 : 2

E. 3 : 2

 

Q. 90 What will come in place of question mark (?) in the given questions?

41 42 45 54 81 ?

A. 194

B. 162

C. 134

D. 146

E. 112

 

Q. 91 What will come in place of question mark (?) in the given questions?

350 334 366 318 ? 302

A. 382

B. 395

C. 376

D. 354

E. 422

 

Q. 92 What will come in place of question mark (?) in the given questions?

164 84 44 24 14 ?

A. 7

B. 4

C. 5

D. 9

E. 6

 

Q. 93 What will come in place of question mark (?) in the given questions?

12 ? 6 9 18 45

A. 5

B. 6

C. 2

D. 4

E. 3

 

Q. 94 What will come in place of question mark (?) in the given questions?

5 4 7 20 79 ?

A. 288

B. 394

C. 265

D. 256

E. 192

 

Q. 95 The profit earned by selling an article for Rs. 590 is double what was incurred when the same article was sold for Rs. 475. What would be the selling price of the article, if it is sold at 20% profit?

A. Rs. 432

B. Rs. 436

C. Rs. 424

D. Rs. 415

E. Rs. 445

 

Questions: 96 – 100

Number of members in 5 health clubs during 5 given years.

 

Q. 96 What is the difference between the total number of members in health clubs A and B together in 2007 and that in health club D and E together in 2011?

A. 131

B. 123

C. 121

D. 127

E. 133

 

Q. 97 If 44% of the total numbers of health clubs, A , B and C together in 2008 are males, what is the total number of male members in health clubs A, B and C together in 2008?

A. 155

B. 170

C. 160

D. 165

E. 175

 

Q. 98 The number of members in health club A increased by what percent from 2009 to 2011?

A. 55 %

B. 80 %

C. 65 %

D. 50 %

E. 75 %

 

Q. 99 What is the average number of members in health club B, C and E in 2010?

A. 132

B. 126

C. 122

D. 124

E. 128

 

Q. 100 What is the ratio of the total number of members in health club C in 2007 and 2009 together to that in health club E in the same year together?

A. 17 : 23

B. 19 : 23

C. 19 : 21

D. 21 : 23

E. 17 : 21

 

 

Answer Sheet
Question 1 2 3 4 5 6 7 8 9 10
Answer A C B D C E C B E A
Question 11 12 13 14 15 16 17 18 19 20
Answer C D E A D B B E B D
Question 21 22 23 24 25 26 27 28 29 30
Answer A B A C D C B E D A
Question 31 32 33 34 35 36 37 38 39 40
Answer A A A C D E E D D B
Question 41 42 43 44 45 46 47 48 49 50
Answer D A D E D B D C C D
Question 51 52 53 54 55 56 57 58 59 60
Answer B B D E A A A B A B
Question 61 62 63 64 65 66 67 68 69 70
Answer B B D B A D D E C A
Question 71 72 73 74 75 76 77 78 79 80
Answer E B D A A A E B E B
Question 81 82 83 84 85 86 87 88 89 90
Answer A A C E A D B E A B
Question 91 92 93 94 95 96 97 98 99 100
Answer A D B B A C D E A A

SBI PO 2017 Prelims Previous Year Paper

SBI PO 2017 Prelims

Section

Questions

Marks

English

30 Questions (1 – 30)

30

Logical Reasoning

35 Questions (31 – 65)

35

Quantitative Aptitude

25 Questions

25

Data Interpretation

10 Questions

10

Q. 1 Which of the phrases given against the sentences should replace the word/phrase given in quotes in the sentence to make it grammatically correct? If the sentence is correct as it is given and no correction is required, select ‘No correction required’ as the answer. The presence of new players in the basketball team is “to additionally attract” for the audience.

A. with add attraction

B. for adding attractive

C. an added attraction

D. of adding attraction

E. No correction required

 

Q. 2 Which of the phrases given against the sentences should replace the word/phrase given in quotes in the sentence to make it grammatically correct? If the sentence is correct as it is given and no correction is required, select ‘No correction required’ as the answer. The dire need of amusement “to escaping boredom” made him cultivate various hobbies.

A. to escape boredom

B. as escaping boredom

C. escapes bored

D. for escape being bored

E. No correction required

 

Q. 3 Which of the phrases given against the sentences should replace the word/phrase given in qoutes in the sentence to make it grammatically correct? If the sentence is correct as it is given and no correction is required, select ‘No correction required’ as the answer. We were credibly informed that the conman has “gave himself” to the police.

A. given over

B. given himself in

C. given himself over

D. given himself up

E. No correction required

 

Q. 4 Which of the phrases given against the sentences should replace the word/phrase given in qoutes in the sentence to make it grammatically correct? If the sentence is correct as it is given and no correction is required, select ‘No correction required’ as the answer. We tempted Karen with many promises but nothing would “worked her up”.

A. working with her

B. works her over

C. works upon her

D. work on her

E. No correction required

 

Q. 5 Which of the phrases given against the sentences should replace the word/phrase given in qoutes in the sentence to make it grammatically correct? If the sentence is correct as it is given and no correction is required, select ‘No correction required’ as the answer. The soil of India “saw growths” of one of the oldest cultures in the world that is the Harappan Civilisation.

A. seen the growth

B. saw the growth

C. had saw growing

D. see the growths

E. No correction required

 

Q. 6 Which of the phrases given against the sentences should replace the word/phrase given in qoutes in the sentence to make it grammatically correct? If the sentence is correct as it is given and no correction is required, select ‘No correction required’ as the answer. A renowned organisation has “recent appointed” a highly acclaimed scientist to head new research and development assistants.

A. recently appointed

B. a recent appointed

C. is to appoint

D. to recently appointed an

E. No correction required

 

Q. 7 Which of the phrases given against the sentences should replace the word/phrase given in qoutes in the sentence to make it grammatically correct? If the sentence is correct as it is given and no correction is required, select ‘No correction required’ as the answer. The serene lush green slope of the hill station “make it up ideal” venue for the meditation camp.

A. making so ideal

B. is ideal for

C. makes it an ideal

D. as of ideal

E. No correction required

 

Q. 8 Which of the phrases given against the sentences should replace the word/phrase given in qoutes in the sentence to make it grammatically correct? If the sentence is correct as it is given and no correction is required, select ‘No correction required’ as the answer. Taking good care of yourself is paramount “for succession” of your goals.

A. about being a success

B. to the success

C. about being successful

D. to succeed

E. No correction required

 

Q. 9 Which of the phrases given against the sentences should replace the word/phrase given in qoutes in the sentence to make it grammatically correct? If the sentence is correct as it is given and no correction is required, select ‘No correction required’ as the answer. The tambourine “to gain popularity” in the mid-18th century in Western Europe as an orchestral instrument.

A. was been popular

B. have popularity

C. has been popular

D. gained popularity

E. No correction required

 

Q. 10 Which of the phrases given against the sentences should replace the word/phrase given in qoutes in the sentence to make it grammatically correct? If the sentence is correct as it is given and no correction is required, select ‘No correction required’ as the answer. Varun advised Aman that “give off” wrong pleasure is not self-sacrifice but self-culture.

A. gave up

B. gives away

C. giving through

D. giving up

E. No correction required

 

Questions: 11 – 20

Read the following passage and answer the given questions. Certain words/phrases are given in quotes to help you locate them while answering some of the questions. There is a market failure in cyber security. Solutions being suggested or “tried” include increasing transparency about data losses, helping consumers and firms to make more informed decisions about cyber security; shedding more light on how Internet Service Providers (ISPs) tackle malware infections they spot on customers’ computers; and using liability laws to force software companies to produce safer code. In transparency, America has led the way. Almost all American states now have data-breach laws that require firms to reveal any loss of sensitive customer information. In Europe telecom firms have been obliged to notify customers of breaches for some time now, and there are plans to extend reporting to a wider range of industries. Breach laws have ecouraged insurance companies to offer coverage against potential losses. This is helpful because they are in a position to gather and share information about best practices across a wide range of companies. A cyberinsurer advises companies on defensive tactics, and also in how to minimise the damage if something goes wrong. The American government should create a cyber-equivalent of the National Transportation Safety Board, which investigates serious accidents and shares information about them. Such a body could look into all breaches that cost over $50m and make sure the lessons are shared widely. But insurers are likely to remain wary of taking on “broader” risks because the costs associated with a “serious” cyber-incident could be astronomic. Insurers can deal with acts of God, but not with acts of Anonymous (hacking groups or acts of state-sponsored hacking). This explains why the overall cyber-insurance market is still small. Governments are weighing in too, not least by supporting private-sector efforts to clean up ‘botnets’, or networks of compromised computers controlled by hackers. These networks, which are prevalent in countries such as America and China, can be used to launch attacks and spread malware. In Germany an initiative called Bot-Frei, which helps people clean up heir infected computers, received government support to get started, though it is now self-financing. The American government has also worked closely with private firms to bring down large botnets. Another strategy involves issuing standards to encourage improved security. America’s National Institute of Standards and Technology published a set of voluntary guidelines for companies in critical infrastructure sectors such as energy and transport. Britain has also launched a scheme called ‘cyber-essentials’ under which firms can apply for a certificate showing they comply with certain minimum security standards. Applicants undergo an external audit and, if successful, are awarded a badge which they can use on marketing materials. Whether governments are best placed to set minimum standards is debatable, but they have certainly raised awarenes of cyber security as an issue that needs attention. They could also help to get more information into the public domain. Researchers have argued persuasively that collecting and publishing data about the quantity of spam and other bad traffic handled by ISPs could encourage the worst performers to do more to tackle the problem, thus improving overall security. Another debate has revolved around getting software companies to produce code with fewer flaws in it. One idea is to make them liable for damage caused when, say, hackers exploit a weakness in a software program. Most software companies currently insist customers accept end-user licensing agreements that specifically protect firms from legal claims unless local laws prohibit such exclusions. The snag is that imposing blanket liability could have a “chilling” effect on innovation. Companies that are selling millions of copies of programmes might take fright at the potential exposure and leave the business. Strict liability be applied only to firms which produce software that cannot be patched if a security flaw is found. There is quite a lot of that sort of code around.

 

Q. 11 Which of the following is the SAME in meaning as the word BROADER as used in the passage?

A. spacious

B. subtle

C. weaker

D. comprehensive

E. approximate

 

Q. 12 Which of the following is/are the argument(s) in favour of cyber-essentials?

A) It boosts transparency and promotion of firms.

B) The certification is given by hackers which makes it authentic.

C) Firms benefit from paying attention to cyber-security and so do users.

A. Only (A)

B. Only (B)

C. Only (A) & (C)

D. Only (B) & (C)

E. All (A), (B) & (C)

 

Q. 13 Which of the following is the SAME in meaning as the word TRIED as used in the passage?

A. convicted

B. accused

C. attempted

D. exasperated

E. None of the given options

 

Q. 14 Which of the following is the OPPOSITE of the word SERIOUS as used in the passage?

A. genuine

B. witty

C. noisy

D. insignificant

E. irresistible

 

Q. 15 Which of the following is the OPPOSITE of the word CHILLING as used in the passage?

A. promoting

B. reassuring

C. encouraging

D. fostering

E. All the given options

 

Q. 16 Which of the following best describes the author’s view of liability laws?

A. These will act as incentives for computer firms to produce more secure software.

B. These are pointless as they cannot be uniformly or strictly implemented.

C. These will not greatly impact computer firms as the financial profits from software are huge.

D. These are not an appropriate approach to cyber security.

E. None of the given options.

 

Q. 17 Which of the following can be said about government’s efforts with regards to cyber security?

A) Government’s efforts have been coupled with private sector co-operation.

B) Government’s efforts have been focused on destroying botnet infrastructure.

C) These are not worthwhile and too small in magnitude.

A. Only (A)

B. Only (B)

C. Only (B) and (C)

D. Only (C)

E. Only (A) and (B)

 

Q. 18 Why has the author mentioned the National Transportation Safety Board in the passage?

A) To urge America to set up a body to share data in cyber-related instances.

B) To monitor cyber security episodes whose losses are over a certain sum.

C) To publish and enforce standards for cyber security for sectors like energy.

A. Only (A)

B. Only (A) and (B)

C. Only (B)

D. Only (B) and (C)

E. All (A), (B) and (C)

 

Q. 19 Which of the following is/are (the) theme(s) of the passage?

A. Holding cyber firms accountable for flaws in their products

B. Cyber-crime infrastructure in certain countries

C. Ways to secure cyber-space

D. Limits of cyber-insurance

E. All the given options are themes

 

Q. 20 Which of the following is/are true in the context of the passage?

A. Breach laws can be helpful for organisations.

B. America is leading the way in terms of laws for disclosure of cyber-beaches

C. Pressure is increasing in software companies to produce safer products

D. Varied efforts are being made to create a market which values cyber-security

E. All the given options are true in the context of the passage

 

Questions: 21 – 30

In the given passage there are words highlighted in quotes. You have to decide if the word is correct (in terms of grammar and context). If not, find out the appropriate word/group of words from the given options. In case, the suggested word is correct, mark the option ‘The given word is correct’ as your answer. Everyone knows that (1) “sustenance” brainboxes is good for the economy. In Thailand, school reformers have an extra incentive to narrow (2) “contrasting” between rich people in cities and their poorer rural cousins, which have (3) “fulfilled” to a decade of political tension and occasional eruption of violence. For years shoddy teaching has favored urban children whose parents can afford to send them to cramming schools or to study abroad. Dismal instruction in the countryside has made it easier for city slickers from posh colleges to paint their political opponents as pliable bumpkins. The dangerous social divide is all the more reason to (4) “worry” about Thailand’s poor rating in an educational league table published in December. Thailand limped into the bottom quarter of 70 countries whose pupils participated in the mathematics, reading and science tests organised under the Programme for International Student Assessment (PISA). It’s scores have (5) “crushed” since a previous assessment in 2012, which researchers found that almost one-third of the country’s 15-year-olds were ‘functionally illiterate’, including almost half of those studying in rural schools. Thailan d’s (6) “error” performance is not dramatically out of step with countries of similar incomes. But it is strange given its usually generous (7) “allocation” on education, which in some years have hoovered up more than a quarter of the  budget. Rote learning is common. There is a shortage of maths and science teachers, but a (8) “overflow” of physical-education instructors. Many head teachers lack the authority to hire or fire their own staff. A big problem is that Thailand spends too much money propping up small schools, where teaching is the poorest. Almost half of Thai schools have fewer than 120 students, and most of those have less than one teacher per class. Opening lots of village schools once helped Thailand (9) “achieve” impressive attendance rates, but road-building and other improvements in infrastructure mean most schools are now within 20 minutes of another. Over the next ten years falling birth rates will reduce school rolls by more than 1m, making it even more (10) “fulfilled” for tiny institutions to provide adequate instruction at a reasonable cost.

 

Q. 21 Which of the following words should be used to correct the quoted words in the passage?

A. diluting

B. nurturing

C. alleviating

D. ornamental

E. The given word is correct

 

Q. 22 Which of the following words should be used to correct the quoted words in the passage?

A. contracts

B. characteristic

C. distinct

D. differences

E. The given word is correct

 

Q. 23 Which of the following words should be used to correct the quoted words in the passage?

A. preceded

B. managed

C. compelled

D. led

E. The given word is correct

 

Q. 24 Which of the following words should be used to correct the quoted words in the passage?

A. concern

B. apprehension

C. fearful

D. distress

E. The given word is correct

 

Q. 25 Which of the following words should be used to correct the quoted words in the passage?

A. deteriorated

B. sink

C. decomposed

D. declining

E. The given word is correct

 

Q. 26 Which of the following words should be used to correct the quoted words in the passage?

A. extreme

B. cheerful

C. dismal

D. inauspicious

E. The given word is correct

 

Q. 27 Which of the following words should be used to correct the quoted words in the passage?

A. employed

B. investing

C. spending

D. setting

E. The given word is correct

 

Q. 28 Which of the following words should be used to correct the quoted words in the passage?

A. satiety

B. surplus with

C. saturated

D. surfeit

E. The given word is correct

 

Q. 29 Which of the following words should be used to correct the quoted words in the passage?

A. complete

B. conclude

C. acquired

D. adhere

E. The given word is correct

 

Q. 30 Which of the following words should be used to correct the quoted words in the passage?

A. difficult

B. ambition

C. troubling

D. doubtful

E. The given word is correct

 

Q. 31 Five movies D, E, F, G and H are released on five different days of the same week starting from Monday and ending on Friday, but not necessarily in the same order. F is released on one of the days before Thursday. Only two movies are released between F and G. H is released immediately before G. D is released on one of the days after H. Which movie is released on Wednesday?

A. E

B. Either G or D

C. G

D. H

E. F

 

Q. 32 Four cartons A, B, Y and Z are placed above one another but not necessarily in the same order. Each carton contains a different drink – Pepsi, Coffee, Frooti and Milkshake – but not necessarily in the same order. Only carton B is kept between the cartons of Pepsi and Frooti. The carton of Coffee is kept immediately below carton of Frooti. The carton of Coffee is kept at one position below Z. What is the position of the carton of Milkshake in the stack?

A. Cannot be determined

B. Immediately below the carton of Frooti

C. First from the bottom

D. First from the top

E. Immediately above the carton of Coffee

 

Questions: 33 – 35

Study the following information and answer the given questions.

Each of the six stores P, Q, R, S, T and U sold different number of books in one day. Only three stores sold less books than U. P sold more books than R. T did not sell the highest number of books. S sold more books than R and P but less than U. The store which sold the second highest number of books sold 72 books.

 

Q. 33 How many books did Q probably sell?

A. 43

B. 58

C. 71

D. 65

E. 89

 

Q. 34 Which of the following stores sold the second lowest number of books?

A. T

B. P

C. S

D. R

E. Q

 

Q. 35 If the total number of books sold by P and T is 125, then how many books did P sell?

A. 51

B. 76

C. 68

D. 45

E. 53

 

Q. 36 In a vertical queue of 13 people, all facing north, K stands exactly at the centre of the queue. No one stands between K and W. Only five people stand between W and P. L stands at one of the positions before P but not at the beginning of the queue. How many people stood after W?

A. Three

B. None

C. Five

D. Can’t be determined

E. Seven

 

Q. 37 In which of the following expressions, does the expression ‘ C

A. P ≥ A ≥ L ≤ E ; C < L ≥ O > N

B. P < A ≤ L ≥ E ; C ≥ L ≤ O < N

C. P = A > L = E ; C = L > O < N

D. P > A > L > E ; C < L < O < N

E. P = A ≥ L < E ; C < L ≥ O ≥ N

 

Q. 38 A person starts from Point A, walks 30m towards south and reaches Point B. He then takes a right turn and walks 7m, he takes a right turn, and walks for 6m. He then takes a right turn and walks 7m. He takes a final left turn, walks a certain distance and reaches Point R. Point R is 17m to the north of Point B. What is the distance between Point A and Point R?

A. 18m

B. 23m

C. 21m

D. 27m

E. 13m

 

Q. 39 S is the only son of V. V is married to R. M is daughter of R. R is grandmother of A. How is S definitely related to A?

A. Uncle

B. Cannot be determined

C. Father

D. Son-in-law

E. Father-in-law

 

Q. 40 What should come in place of S and # respectively in the expression :

P > A$R < O < T;S < L < A#M, so that the expression T>M definitely holds true?

A. >,<

B. <,=

C. <,<

D. <, ≤

E. >,>

 

Questions: 41 – 45

Study the following information and answer the given questions.

C, D, E, F, W, X, Y and Z have to attend a wedding in January, April, September and December of the same year. In each month the wedding is on either the 11th or on 24th of the month. Not more than two of the given people have to attend a wedding in the same month. W has to attend a wedding on 11th of the month which has only 30 days. Only three people have to attend a wedding between W and Y. C and Y have to attend a wedding neither on the same date nor in the same month. C does not have to attend a wedding in April. Only two people have to attend a wedding between C and F. X and F have to attend a wedding on the same date. D has to attend a wedding in one of the days before X. Only one person has to attend a wedding between D and E. Less than four people have to attend a wedding between E and Z.

 

Q. 41 How many people have to attend a wedding between F and Z?

A. Two

B. Three

C. None

D. More than three

E. One

 

Q. 42 When does X have to attend a wedding?

A. 24th April

B. Cannot be determined

C. 11th January

D. 24th September

E. 11th December

 

Q. 43 If so the people are made to attend the wedding in alphabetical order starting from 11th January and ending in 24th December, then the schedule of how many people will remain unchanged?

A. One

B. Two

C. Five

D. None

E. Three

 

Q. 44 Who among the following has/have to attend a wedding before Y?

A. Both C and X

B. Only W

C. None

D. Both F and W

E. Only F

 

Q. 45 As per the given arrangement, four of the following five are alike in a certain way and so form a group. Which of the following does not belong to the group?

A. W

B. F

C. Z

D. Y

E. X

 

Q. 46 In the question, relationship between different elements is shown in the statements. The statements are followed by two conclusions. Study the conclusions based on the given statements and select the appropriate answer.

Statement:

M < O < U ≤ R≤ T

P ≥ R ≤ I ≤ C < L

Conclusions:

I. L > M

II. O ≤ C

A. Either conclusion I or II is true

B. Neither conclusion I nor II is true

C. Only conclusion II is true

D. Both conclusion I and II are true

E. Only conclusion I is true

 

Q. 47 In the question, relationship between different elements is shown in the statements. The statements are followed by two conclusions. Study the conclusions based on the given statements and select the appropriate answer.

Statements:

C < L = I ≤ N > G

I < M ≥ O > R > T

Conclusions:

I. C < O

II. G > T

A. Neither conclusion I nor II is true

B. Only conclusion I is true

C. Both conclusion I and II are true

D. Only conclusion II is true

E. Either conclusion I or II is true

 

Q. 48 In the question, relationship between different elements is shown in the statements. The statements are followed by two conclusions. Study the conclusions based on the given statements and select the appropriate answer.

Statements:

M < O ≤ U ≤ R ≥ T

P ≥ R ≤ I ≤ C < L

Conclusions:

I. P > T

II. P = T

A. Only conclusion I is true

B. Neither conclusion I nor II is true

C. Both conclusion I and II are true

D. Only conclusion II is true

E. Either conclusion I or II is true

 

Questions: 49 – 53

Read the given information to answer the given questions.

Eight people X, G, T, C, P, J, A and M live on different floors of a building. The ground floor of the building is numbered one, the one above that is numbered two and so on till the topmost floor is numbered eight. All of them can perform a different form of dance- Kathak, Garba, Dandiya, Bhangra, Lavani, Odissi, Mohiniyattam and Sattriya.

(Note: None of the given information is necessarily in the same order.)

T lives on an even numbered floor below floor number five. Only three people live between T and the one who performs Garba. As many people live below T as above the ones who perform Lavani. The number of people living between the ones who perform Garba and Lavani is equal to the number of people living between C and P. C lives on an even-numbered floor above P. Neither C nor P performs Garba or Lavani. The one who performs Kathak lives on an oddnumbered floor below floor number four. P does not perform Kathak. The number of people between T and the one who performs Kathak is same as the number of people living between X and the one who performs Lavani. X lives on  one of the floors above the one who performs Lavani. X lives on one of the floors above the one who performs Lavani. The number of people living between C and X equal to the number of people living between T and M. The one who performs Odissi lives on an even-numbered floor immediately above the one who performs Bhangra. Only three people live between G and J. G lives in one of the floors above J. The one who performs Dandiya lives immediately above the one who Orient’s Sattriya.

 

Q. 49 Which of the following is true as per the given arrangement?

A. C performs Odissi.

B. The one who performs Garba lives on floor number six.

C. T lives immediately above J.

D. None of the given statements is true.

E. Only two people live betweeen M and G.

 

Q. 50 Four of the following five are alike in a certain way based on the given arrangement and thus form a group. Which of the following does not belong to the group?

A. M-Bhangra

B. C-Dandiya

C. P-Odissi

D. G-Floor number eight

E. J-Floor number two

 

Q. 51 How many people live between the ones who perform Lavani and Bhangra?

A. More than three

B. Three

C. None

D. One

E. Two

 

Q. 52 Who performs Sattriya?

A. G

B. A

C. P

D. T

E. C

 

Q. 53 Which dance does M perform?

A. Bhangra

B. Lavani

C. Odissi

D. Mohiniyattam

E. Kathak

 

Questions: 54 – 58

Study the given information carefully to answer the given questions.

Eight people A, B, C, D, E, F, G and H were born in different years, viz, 1961, 1970, 1974, 1980, 1983, 1987, 1996 and 2000 but not necessarily in the same order.

Note: (A) All calculations are done with respect to the present year, 2017, assuming the month and date to be the same as that of the years of birth as mentioned above.

(B) Each person is assumed to be born on the same date and same month of the respective years. D was born after 1983 but not in the year 2000. The sum of the present ages of A and D is 64. The difference between the present ages of A and G is less than 5. B was born in an odd-numbered year. B is older than G. The sum of the present ages of F and C is 68. F is younger than C. H is not the youngest.

 

Q. 54 Four of the following five are alike in a certain way as per the given arrangement and hence form a group. Which of the following does not belong to the group?

A. E

B. B

C. C

D. F

E. G

 

Q. 55 How many persons are younger than E?

A. Two

B. More than three

C. None

D. Three

E. One

 

Q. 56 Who amongst the following was born in the year 1996?

A. E

B. C

C. A

D. G

E. None of these

 

Q. 57 If A’s uncle is 21 years older than A, then how old is A’s uncle at present (in years)?

A. 77

B. 68

C. 89

D. 64

E. 55

 

Q. 58 Which of the following represents the difference between the present ages of B and H (in years)?

A. 22

B. 9

C. 25

D. 32

E. 13

 

Q. 59 In the number 7853921, one is added to each digit exactly divisible by two. Two is subtracted from each digit exactly divisible by three. All other digits are kept unchanged. In the new number thus formed, which if the following digits will appear twice?

A. None

B. Only 5

C. Both 2 and 3

D. Only 3

E. Both 1 and 7

 

Q. 60 In a certain code language, ‘job requires expertise’ is written as ‘la nu si’, ‘expertise in area’ is written as ‘li bo la’ and ‘requires area inspection’ written as ‘si dm bo’. How is ‘inspection’ written in that code language? (All the given codes are two-letter codes only)

A. si

B. Either ‘nu’ or ‘si’

C. Either ‘bo’ or ‘si’

D. dm

E. bo

 

Questions: 61 – 65

Study the following information to answer the given questions.

Eight persons F, G, H, I, O, P, Q and R are seated in a straight line facing north. Each of them works on a different floor of an office building, viz, 7th, 16th, 18th, 23rd, 31st, 35th, 44th and 47th. None of the given information is necessarily in the same order.

● O sits fourth to the right of the one who works on the 31st floor. The one who works on the 23rd floor sits second to the right of O.

● Q sits third to the left of I. I is not an immediate neighbor of O. Q does not sit at any of the extreme ends of the line.

● Only two people sit between Q and P. The one who works on the 44th floor sits on the immediate right of the H. H is not an immediate neighbor of P.

● The difference between the numerical values of the floor numbers on which P and the one on the immediate right of P work is 13.

● Only one person sits between F and the one who works on the 35th floor. F is not an immediate neighbor of I.

● More than two people sit between R and the one who works on the 16th floor. O does not work on the 16th floor.

● H works on a floor lower than O.

 

Q. 61 Which of the following pairs represents the persons seated at the two extreme ends of the line?

A. G and the one working on the 7th floor

B. I, R

C. The ones working on the 18th and 44th floors

D. G, R

E. The one working on 23rd floor and P

 

Q. 62 What is the difference between the floor numbers on which P and R work?

A. 31

B. 4

C. 3

D. 16

E. 15

 

Q. 63 F is related to the one who works on the 47th floor following a certain pattern based on the given arrangement. In the same pattern, P is related to the one who works on the 44th floor. Who amongst the following is H related to following the same pattern?

A. The one who works on the 35th floor

B. The one who is on the immediate left of R

C. The one who is sitting second to the left of O

D. The one who works on the 16th floor

E. O

 

Q. 64 Fill in the blanks (respectively in the same order) in order to make the statement correct based on the given arrangement.

G __________ and Q __________.

A. works on the 47th floor, sits to the immediate left of Q.

B. works in one of the floors above H, works in one of the floors below F.

C. sits on the immediate left of I, works on the 44th floor.

D. sits second to the right of O, work three floors above O.

E. Other than those given as options

 

Q. 65 How many people sit on the left of the one who works on the 35th floor?

A. One

B. Two

C. None

D. Four

E. Three

 

Q. 66 What approximate value will come in place of X in the following equation?

X² – 137.99 ÷ 6 = 21.99 × 23.01

A. 23

B. 50

C. 42

D. 29

E. 35

 

Q. 67 What approximate value will come in place of X in the following equation?

X% of 400.02 + 12.93² = 285

A. 18

B. 15

C. 24

D. 34

E. 29

 

Q. 68 What approximate value will come in place of X in the following equation?

(3327.99 – 27.93) ÷ X = 110 × 5.99

A. 9

B. 1

C. 19

D. 15

E. 5

 

Q. 69 What approximate value will come in place of X in the following equation?

5520 ÷ 12.01 + √226 × 5.99 = X

A. 350

B. 550

C. 500

D. 450

E. 250

 

Q. 70 What approximate value will come in place of X in the following equation?

160.01 + 40 ÷ (16.5 ÷ 33) = ?

A. 310

B. 290

C. 250

D. 350

E. 240

 

Questions: 71 – 75

Refer to the graph and answer the given questions.

 

Q. 71 The number of students who opted for Course B in 2013 was what percent more than who opted for course A in 2013?

A. 45+1/6

B. 42+4/9

C. 38+8/9

D. 56+7/9

E. 62+1/9

 

Q. 72 What is the difference between the total number of students who opted for Courses A and B together in 2012 and that who opted for both the courses together in 2014?

A. 50

B. 30

C. 60

D. 40

E. None of these

 

Q. 73 In 2014, if ‘X’ students passed Course A and B each and the ratio of the number of students who failed in Courses A and B respective was 5 : 2, what is the value of ‘X’?

A. 190

B. 220

C. 160

D. 150

E. 180

 

Q. 74 What is the average number of students who opted for Course A in 2010, 2011 and 2012?

A. 225

B. 250

C. 230

D. 240

E. 260

 

Q. 75 The number of students who opted for Courses A and B in 2011 was respectively 25% more and 35% less than that in 2009. What was the total number of students who opted for Courses A and B together in 2009?

A. 600

B. 540

C. 575

D. 560

E. 650

 

Q. 76 The ratio of the numerical values of curved surface area to the volume of the right circular cylinder is 1:7. If the ratio of the diameter to the height of the cylinder is 7:5, then what is the total surface area of the cylinder? (in m²)

A. 2992

B. 3172

C. 2882

D. 3576

E. 3992

 

Q. 77 The time taken by a boat to cover a distance of ‘D-56’ km upstream is half of that taken by it to cover a distance of ‘D’ km downstream. The ratio of the speed of the boat downstream to that upstream is 5 : 3. If the time taken to cover ‘D-32’ km upstream is 4 hours, what is the speed of water current? (in km/h)

A. 5

B. 3

C. 4

D. 16

E. 8

 

Q. 78 Poona invests ₹4200 in Scheme A, which offers 12% p.a. simple interest. She also invested ₹(4200 – P) in Scheme B offering 10% p.a. compound interest (compounded annually). The difference between the interests Poona earned from both the schemes at the end of 2 years is ₹294. What is the value of P?

A. 1500

B. 800

C. 600

D. 1000

E. Other than those given as options

 

Q. 79 A man sold two articles – A (at a profit of 40%) and B (at a loss of 20%). He earned a total profit of ₹8 in the whole deal. If article A costs ₹140 less than article B, what is the price of article B?

A. ₹380

B. ₹280

C. ₹340

D. ₹375

E. ₹300

 

Q. 80 B is eighteen years younger than A. The ratio of B’s age six years hence to C’s present age is 3 : 2. If at present A’s age is twice the age of C, then what was B’s age 4 years ago?

A. 24 years

B. 28 years

C. 26 years

D. 20 years

E. 16 years

 

Q. 81 The time taken by A alone to finish a piece of work is 60% more than that taken by A and B together to finish the same piece of work. C is twice as efficient as B. If B and together can complete the same piece of work in 13+1/3 days, in how many days can A alone finish the same piece of work?

A. 32

B. 24

C. 16

D. 28

E. Other than those given as options

 

Q. 82 Out of her monthly salary, Ridhi spends 34% on various expenses. From the remaining, she gives 1/6th to her brother, 2/3 to her sister and the remaining she keeps as savings. If the difference between the amounts she gave to her sister and brother was ₹10,560, what was Ridhi’s savings?

A. ₹3740

B. ₹3420

C. ₹4230

D. ₹3230

E. Other than those given as options

 

Q. 83 A bag contains 63 cards (numbered 1,2,3…63). Two cards are picked at random from the bag (one after another and without replacement). What is the probability that the sum of the numbers of both the cards drawn is even?

A. 11/21

B. 34/63

C. 7/11

D. 11/63

E. Other than those given as options

 

Q. 84 A, B and C started a business and invested in the ratio of 3 : 4 : 5. After 4 months A withdrew 1/12 of the amount of what B and C had invested. If the annual income was ₹9200 then what was the share of B?

A. ₹4200

B. ₹3800

C. ₹3600

D. ₹2800

E. Other than the given options

 

Q. 85 In each question, two equations numbered I and II are given. You have to solve both the equations and mark the appropriate answer.

I. 3x² – 4x + 1 = 0

II. 15y² – 8y + 1 = 0

A. x > y

B. x ≤ y

C. x = y or relation can’t be established

D. x ≥ y

E. x < y

 

Q. 86 In each question, two equations numbered I and II are given. You have to solve both the equations and mark the appropriate answer.

I. x² + 14x + 45 = 0

II. y² + 19y + 88 = 0

A. x ≥ y

B. x < y

C. x ≤ y

D. x = y or relation can’t be established

E. x > y

 

Q. 87 In each question, two equations numbered I and II are given. You have to solve both the equations and mark the appropriate answer.

I. x² – 2x – 8 = 0

II. y² + 15y + 54 = 0

A. x = y or relation can’t be established

B. x ≤ y

C. x > y

D. x ≥ y

E. x < y

 

Q. 88 In each question, two equations numbered I and II are given. You have to solve both the equations and mark the appropriate answer.

I. 2x² – 9x +9 = 0

II. y² – 7y + 12 = 0

A. x < y

B. x ≤ y

C. x > y

D. x = y or relation can’t be established

E. x ≥ y

 

Q. 89 In each question, two equations numbered I and II are given. You have to solve both the equations and mark the appropriate answer.

I. x² = 121

II. y² – 23y + 132 = 0

A. x > y

B. x ≥ y

C. x < y

D. x ≤ y

E. x = y or relation can’t be established

 

Q. 90 A jar contains a mixture of milk and water in the ratio of 3:1. Now, 1/25 of the mixture is taken out and 24 litres water is added to it. If the resultant ratio of milk to water in the jar was 2:1, what was the initial quantity of mixture in the jar? (in litres)

A. 160

B. 180

C. 200

D. 250

E. Other than the given options

 

Q. 91 What will come in place of X in the then number series?

“17”, “9”, “10”, “16.5”, “X”, “90”

A. 44

B. 35

C. 48

D. 38

E. 33

 

Q. 92 What will come in place of X in the then number series?

“7”, “6”, “10”, “27”, “X”, “515”

A. 112

B. 104

C. 114

D. 96

E. 108

 

Q. 93 What will come in place of X in the then number series?

“33”, “40”, “29”, “42”, “25”, “X”

A. 40

B. 44

C. 52

D. 48

E. 46

 

Q. 94 What will come in place of X in the then number series?

“316”, “307”, “282”, “233”, “152”, “X”

A. 35

B. 25

C. 31

D. 41

E. 47

 

Q. 95 What will come in place of X in the then number series?

“5”, “9”, “33”, “72”, “121”, “X”

A. 169

B. 163

C. 171

D. 184

E. Other than those given as options

 

Questions: 96 – 100

Study the table and answer the given questions.

 

Q. 96 What is the average number of people who voted at centres B, D and E?

A. 1700

B. 1628

C. 1720

D. 1740

E. 1560

 

Q. 97 What percent of the total number of registered voters cast invalid votes at Centre D, if the number of invalid votes cast at Centre D was 10% of the number of votes cast?

A. 5.5

B. 8.5

C. 7.6

D. 6.5

E. Other than those given as options

 

Q. 98 At centres F, the total number of registered voters was 25% less than that at Centre C. At Centre F, the number of people who voted was 450 less than that at Centre C and 150 votes cast were declared invalid. What was the ratio of the number of valid votes cast to the total number of registered voters at Centre F?

A. 4 : 5

B. 3 : 4

C. 2 : 3

D. 6 : 1

E. 5 : 8

 

Q. 99 The number of people who did not vote at Centre D was what percent more than that who did not vote at Centre A?

A. 42+6/7

B. 35+2/3

C. 37+1/7

D. 43+5/8

E. Other than those given as options

 

Q. 100 What is the difference between the total number of people who did not vote at Centres A and B together and that who did not vote at Centre D and E together?

A. 80

B. 60

C. 50

D. 70

E. 116

 

 

Answer Sheet
Question 1 2 3 4 5 6 7 8 9 10
Answer C A B D C A C B D D
Question 11 12 13 14 15 16 17 18 19 20
Answer D C C D E D E B E E
Question 21 22 23 24 25 26 27 28 29 30
Answer B D D E A C C D B A
Question 31 32 33 34 35 36 37 38 39 40
Answer A A B E E B E C B D
Question 41 42 43 44 45 46 47 48 49 50
Answer E E C C E A E E D D
Question 51 52 53 54 55 56 57 58 59 60
Answer E D B C E E E E E D
Question 61 62 63 64 65 66 67 68 69 70
Answer B B D C C A E E B E
Question 71 72 73 74 75 76 77 78 79 80
Answer C E B D D A C B C C
Question 81 82 83 84 85 86 87 88 89 90
Answer B E E E D D C B D C
Question 91 92 93 94 95 96 97 98 99 100
Answer B B B C B C C C C E

SBI PO 2016 Prelims Previous Year Paper

SBI PO 2016 Prelims

Section

Questions

Marks

English

30 Questions (1 – 30)

30

Data Interpretation

10 Questions

10

Quantitative Aptitude

25 Questions

25

Logical Reasoning

35 Questions (66 – 100)

35

Q. 1 In the following questions, each sentence has two blanks, each blank indicating that something has been omitted. Choose the set of words for the blanks which best fits the meaning of the sentence as a whole.

By running liquid chocolate ___________ an electric field, researchers were able to make it ________ more easily.

A. across, gushing

B. during, stream

C. through, flow

D. among, move

E. in, tidal

 

Q. 2 In the following questions, each sentence has two blanks, each blank indicating that something has been omitted. Choose the set of words for the blanks which best fits the meaning of the sentence as a whole.

India is ________ a tea – drinking country, with coffee______ to the instant variety, except in the south.

A. essentially, only

B. most, confined

C. simply, limiting

D. predominantly, restricted

E. larger, constrained

 

Q. 3 In the following questions, each sentence has two blanks, each blank indicating that something has been omitted. Choose the set of words for the blanks which best fits the meaning of the sentence as a whole.

There is no______ of capital to finance massive infrastructure development provided involving huge investments are _______ with transparency.

A. dearth, handled

B. deny, managed

C. shortage, completely

D. paucity, direct

E. absent, persuaded

 

Q. 4 In the following questions, each sentence has two blanks, each blank indicating that something has been omitted. Choose the set of words for the blanks which best fits the meaning of the sentence as a whole.

Auto makers are likely to use the cash pool to ______ R&D capabilities as well as ________ new models.

A. restored, launch

B. strengthen, introduce

C. extent, offer

D. enhance, advanced

E. create, ushers

 

Q. 5 In the following questions, each sentence has two blanks, each blank indicating that something has been omitted. Choose the set of words for the blanks which best fits the meaning of the sentence as a whole.

As per the _________norms that remain unchanged firms will have to ________ the sourcing norms from the time of opening of the first store.

A. prevailing, adhere

B. existing, meet

C. recently, accomodate

D. current, abide

E. actually, fit

 

Questions: 6 – 10

Rearrange the given sentences / group of sentences (A), (B), (C), (D), (E) and (F) in proper sequence as to form a meaningful paragraph and then answer the given questions.

(A) It is evident from the fact that from shopping, ordering food, booking vacations to money transfers, everything is going digital.

(B) Technology today has seeped into every facet of our lives.

(C) This and a variety of problems now can be solved easily as healthcare providers can adopt technologies to improve, simplify and better manage the quality of their services.

(D) The first step in this direction would be capturing all patient health information at the point of care, especially hospitals and clinics.

(F) For instance, in medical emergency situations, the lack of readily available patient health information at the point of care still impacts millions of people each year leading to adverse health conditions and even death.

 

Q. 6 Which of the following should be the FIFTH sentence after the rearrangement?

A. E

B. D

C. A

D. F

E. C

 

Q. 7 Which of the following should be the SECOND sentence after the rearrangement?

A. A

B. B

C. F

D. D

E. C

 

Q. 8 Which of the following should be the SIXTH (LAST) sentence after the rearrangement?

A. E

B. D

C. A

D. B

E. F

 

Q. 9 Which of the following should be the FIRST sentence after the rearrangement?

A. A

B. C

C. B

D. F

E. E

 

Q. 10 Which of the following should be the FOURTH sentence after the rearrangement?

A. A

B. B

C. C

D. F

E. D

 

Questions: 11 – 20

Read the following passage carefully and answer the questions given below it. Certain words/phrases have been given in bold to help you locate them while answering some of the questions At first glance the patriarchy appears to be thriving. More than 90% of Presidents and Prime Ministers are male, as are all nearly big corporate bosses. Men dominate finance, technology, films, sports, music and even stand – up comedy. In much of the world they still enjoy social and legal privileges simply because they have a Y chromosome. So it might seem odd to worry about the plight of men. Yet there is plenty of cause for concern. Men cluster at the bottom as well at the top. Poorly educated men in rich countries have had difficulty coping with the enormous changes in the labour market and the home over the past half – century. As technology and trade have devalued brawn, less – educated men have struggled to find a role in the workplace. Women on the other hand, are surging into expanding sectors such as health care and education, helped by their superior skills. As education has become more important, boys have also fallen behind girls in school (except at the very top). Men who lose jobs in manufacturing often never work again. And men without work find it hard to support a family. The result for low – skilled men, is a poisonous combination of no job, no family and no prospects. Some tend to focus on economics. Shrinking job opportunities for men, they say, are entrenching poverty and destroying families. In America pay for men with only a high – school certificate fell by 21% in real terms between 1979 and 2013, for women with similar qualifications it raised by 3%. Around a fifth of working – age American men with only a high – school have no job. But both economic and social changes are to be blame and the two causes reinforce each other. Moreover, these problems are likely to get worse. Technology will disrupt more industries, creating benefits for society but rendering workers who fail to update their skills redundant. The OECD, a think – tank, predicts that the absolute number of single – parent house-holds will continue to rise in nearly all rich countries. Boys who grow up without fathers are more likely to have trouble forming lasting relationships, creating a cycle of male dysfunctional. What can be done? Part of the solution lies in a change in cultural attitudes. Over the past generation, men have learned that they need to help with child care and have changed their behavior. Women have learned that they can be surgeons and physicists not at the cost of motherhood. Policymakers also need to lend a hand, because foolish laws are making the problem worse. Governments need to recognize that boys underachievement is a serious problem and set about fixing it. Some sensible policies that are good for everybody are particularly good for boys. Early childhood education provides boys with more structure and a better chance of developing verbal and social skills. Countries with successful vocational systems such as Germany have done a better job motivating non – academic boys and guiding them into jobs, but policymakers need to reinvent vocational education for an age when trainers are more likely to get jobs in hospitals than factories. The growing equality of the genders is one of the biggest achievements of the post – war era people have greater oppurtunities than ever before to achieve their ambitions regardless of their gender. But some even have failed to cope with this new world. It is time to give them a hand.

 

Q. 11 What do the statistics in the passage with regard to America indicate?

A. Pay and employment for men with low skills has fallen.

B. Less than ten percent of American politicians are women.

C. Men dominate most high paying professions such as surgery etc.

D. Americans are protected from labour market disruptions by a strong social welfare system.

E. At both the top and bottom of the job pyramid women are losing out to men.

 

Q. 12 According to the passage, what can be said about families today?

A. It is difficult for families to survive on the earnings of one parent.

B. The size of the family is shrinking which will cause a problem of an ageing population soon.

C. Parents favour sons over daughters causing an unhealthy social mindset.

D. Poverty and lack of education among men is putting the family structure at risk.

E. None of the given options can be said.

 

Q. 13 Choose the word/group of words which is most nearly the same in meaning as the word ENJOY given in bold as used in the passage.

A. benefit from

B. laugh with

C. amused by

D. prefer to

E. liking to

 

Q. 14 Choose the word/group of words which is opposite in meaning as the word ENTRENCHING given in bold as used in the passage.

A. hastening

B. reducing

C. stablising

D. hesitating

E. digging

 

Q. 15 Which of the following best describes the author’s opinion about vocational education?

A. Germany’s model of vocational guidance need not be adopted by other countries.

B. There should be less focus on vocational skills.

C. Vocational education should equip men with better skills than they do at present.

D. Vocational education for women should be given more financial support.

E. Rather than obtaining an unaffordable university education, vocational education is a practical solution.

 

Q. 16 Choose the word/group of words which is most nearly the same in meaning as the word STRUCTURE given in bold as used in the passage.

A. building

B. order

C. assembly

D. layout

E. procedure

 

Q. 17 Which of the following is true in the context of the passage?

A. Underachievement of boys at school is a problem prevalent in Japan and America.

B. Most of the Government funds for vocational education are misused.

C. Women are still not well represented in high positions.

D. The number of children with learning disabilities is on the rise in rich countries.

E. None of the given statements is true in the context of the passage.

 

Q. 18 Which of the following is an appropriate title for the passage?

A. Flying Too High

B. Beyond Face Value : Men at Risk

C. The Fundamentals of Achievement

D. Poverty Misleading Figures

E. Health, Wealth and the Origins of Inequality

 

Q. 19 Which of the following factors has/have an impact on the current state of male Employment?

(A) Men’s failure to update their skills.

(B) Single parent households with an absent father

(C) Adoption of technological changes in low skill job.

A. Only (A)

B. Only (B)

C. All (A), (B) and (C)

D. Only (A) and (C)

E. Only (A) and (B)

 

Q. 20 Choose the word/group of words which is opposite in meaning as the word SURGING given in bold as used in the passage.

A. flowing

B. delaying

C. precipitating

D. hollowing

E. disappearing

 

Q. 21 Read each sentence to find out whether there is any grammatical error in it. The error, if any, will be in one part of the sentence. Select the part with the error as your answer. If there is no error, select ‘No error’ as your answer. (Ignore the errors of punctuation, if any).

A. The infection is commonly found

B. in pork eaters and is

C. also a result on unhygienic practices

D. like not cleaning vegetables properly.

E. No error

 

Q. 22 Read each sentence to find out whether there is any grammatical error in it. The error, if any, will be in one part of the sentence. Select the part with the error as your answer. If there is no error, select ‘No error’ as your answer. (Ignore the errors of punctuation, if any).

A. One of the greatest

B. surrealists of his time

C. as was the ultimate conquistador

D. in an art he alone perfected.

E. No error

 

Q. 23 Read each sentence to find out whether there is any grammatical error in it. The error, if any, will be in one part of the sentence. Select the part with the error as your answer. If there is no error, select ‘No error’ as your answer. (Ignore the errors of punctuation, if any).

A. In the

B. plush property – a

C. seat in power

D. or a political ill omen?

E. No error

 

Q. 24 Read each sentence to find out whether there is any grammatical error in it. The error, if any, will be in one part of the sentence. Select the part with the error as your answer. If there is no error, select ‘No error’ as your answer. (Ignore the errors of punctuation, if any).

A. A gang war inside

B. the high security jail resulted

C. in four under trials

D. being rushed for the hospital

E. No error

 

Q. 25 Read each sentence to find out whether there is any grammatical error in it. The error, if any, will be in one part of the sentence. Select the part with the error as your answer. If there is no error, select ‘No error’ as your answer. (Ignore the errors of punctuation, if any).

A. The museum suffered

B. severe damaged after

C. fire hydrants were

D. rendered useless.

E. No error

 

Questions: 26 – 30

In the following passage, there are blanks, each of which has been numbered. Against each, five words are suggested, one of which fits the blank appropriately. Find out the appropriate word in each case. Technology has changed society, There is no (26)……. that we live in a world of instant gratification which is driven by the latest advancement in technology and devices. With the Internet, social networks and cell phones the way people communicate has changed(27)…….. We would (28)………….. email than men and instead of talking on the phone we would prefer to text. (29)…….. technology has advanced the access of information and communication : our society has grown more (30)………….. by communicating using technologies, therefore our culture is losing the ability to socialise face to face.

 

Q. 26 ……….

A. doubts

B. denying

C. way

D. questions

E. alarm

 

Q. 27 ……….

A. drastically

B. whole

C. positively

D. cursorily

E. haltingly

 

Q. 28 ……….

A. prefer

B. besides

C. despite

D. instead

E. rather

 

Q. 29 ……….

A. But

B. Then

C. However

D. Although

E. When

 

Q. 30 ………

A. impersonal

B. contacts

C. engaging

D. knitted

E. warm

 

Questions: 31 – 35

 

Q. 31 What is the respective ratio between the total number of students who have enrolled for coaching institutes X and Y together from school R and the total number of students who have enrolled for the same coaching institutes together from school S?

A. 5 : 2

B. 5 : 4

C. 15 : 4

D. 25 : 8

E. 15 : 8

 

Q. 32 In coaching institute W, 40% of students are females. If 5/9 of the total females are from school R, what is the number of male students from school R who have enrolled for coaching institute W?

A. 15

B. 16

C. 14

D. 13

E. 12

 

Q. 33 What is the difference between the total number of students who have enrolled for coaching institute W from schools P and S together and the total number of students who have enrolled for coaching institute Y from the same schools togther?

A. 127

B. 126

C. 136

D. 116

E. None of these

 

Q. 34 What is the average number of students who have enrolled in coaching institutes X, Y and Z from school Q?

A. 63

B. 65

C. 62

D. 64

E. 61

 

Q. 35 In coaching institute Z, the total number of students who have enrolled from schools R and S together is what percent less than the total number of students enrolled from schools P and Q together?

A. 156/5

B. 161/5

C. 167/5

D. 100/3

E. 205/6

 

Q. 36 The respective ratio of radii of two right circular cylinders( A and B) is 4 : 5. The respective ratio of volume of cylinders A and B is 12 : 25. What is the respective ratio of the heights of cylinders A and B?

A. 2 : 3

B. 3 : 5

C. 5 : 8

D. 4 : 5

E. 3 : 4

 

Q. 37 Dhruva gave 35% of her monthly salary to her mother. From the remaining salary, she paid 18% towards rent and 42% she kept aside for her monthly expenses. The remaining amount she kept in bank account. The sum of the amount she kept in bank and that she gave to her mother was Rs. 43,290. What was her monthly salary?

A. Rs. 80,000

B. Rs. 75,000

C. Rs. 64,000

D. Rs. 76000

E. Rs. 72,000

 

Q. 38 18 litres of pure water was added to a vessel containing 80 liters of pure milk. 49 liters of the resultant mixture was then sold and some more quantity of pure milk and pure water was added to the vessel in the respective ratio 2 : 1. If the resultant respective ratio of milk and water in the vessel was 4 : 1, what was the quantity of pure milk added in the vessel? (in liters)

A. 4

B. 8

C. 10

D. 12

E. 2

 

Questions: 39 – 43

 

Q. 39 What is the total number of cars sold in the year 2013, if the growth rate in the total number of cars sold from 2012 to 2013 was the same as that of 2011 to 2012?

A. 23096

B. 210978

C. 24189

D. 26623

E. 24659

 

Q. 40 In which of the following years the ratio of number of Maruti cars sold to the total number of cars sold was lowest?

A. 2011

B. 2006

C. 2009

D. 2008

E. 2010

 

Q. 41 What was the approximate average number of cars sold (other than Maruthi cars) in a year, taking the data of all the years together?

A. 11231

B. 10317

C. 10919

D. 10121

E. 12117

 

Q. 42 The number of Maruthi cars sold in the years 2007, 2008 and 2009 is approximately what percent of total number of all the cars sold in the years 2006, 2007, 2009, 2011 and 2012?

A. 22%

B. 25%

C. 17%

D. 29%

E. 11%

 

Q. 43 What is the ratio between the number of cars sold other than that of Maruti in 2011 to the total number of cars sold in 2006?

A. 1.26

B. 1.17

C. 1.04

D. 1.23

E. 1.62

 

Q. 44 There are two motor cycles (A and B) of equal cost price. Motorcycle A was sold at a profit of 14% and motorcycle B was sold for Rs. 4290 more than its cost price. The net profit earned after selling both the motorcycles (A and B) is 20%. What is the cost price of each motorcycle.

A. Rs. 16,500

B. Rs. 16,000

C. Rs. 15,500

D. Rs. 17,500

E. Rs. 17,000

 

Q. 45 A bag contains 3 white balls and 2 black balls. Another bag contains 2 white and 4 black balls. A bag and a ball are picked at a random. What is the probability that the ball drawn is white?

A. 7/11

B. 7/30

C. 5/11

D. 7/15

E. 1/15

 

Questions: 46 – 50

What will come in place of the question mark (?) in the following series ?

 

Q. 46  16 17 21 30 46 ?

A. 82

B. 104

C. 71

D. 92

E. 84

 

Q. 47  2 2 4 12 48 ?

A. 180

B. 220

C. 240

D. 160

E. 210

 

Q. 48  10 14 23 37 56 ?

A. 74

B. 80

C. 118

D. 120

E. 94

 

Q. 49  4 7 13 25 49 ?

A. 118

B. 136

C. 86

D. 97

E. 124

 

Q. 50  50 59 41 68 32 ?

A. 77

B. 45

C. 20

D. 85

E. 60

 

Q. 51 A is thrice as efficient as B and takes 10 days less to do a piece of work than B takes to do the same work. In how many days, B alone can finish the whole work?

A. 15 days

B. 10 days

C. 9 days

D. 8 days

E. 7 days

 

Q. 52 The compound interest on a certain sum for 2 years at 10% per annum is Rs. 525. The simple interest on the same sum for double the time at half the rate percent per annum is

A. Rs. 400

B. Rs. 500

C. Rs. 600

D. Rs. 800

E. None of these

 

Questions: 53 – 57

What approximate value will come in place of the question mark (?) in the given questions? (You are not expected to calculate the exact value)

 

Q. 53  26.0003 – (154.001/6.995) = ?

A. 4

B. 18

C. 9

D. 10

E. 14

 

Q. 54  (17.995/3.01) + (104.001/12.999) + ?

A. 11

B. 20

C. 23

D. 14

E. 17

 

Q. 55  3/5 of 4/7 of 7/9 of 425 = ?

A. 121

B. 110

C. 118

D. 113

E. 124

 

Q. 56  124.001 x 14.001÷ 3.4999 + 2² = ?

A. 500

B. 450

C. 425

D. 475

E. 550

 

Q. 57  18.0009 ÷ √35.333 × (369 + ?) = 2040.05

A. 302

B. 298

C. 322

D. 319

E. 311

 

Q. 58 A, B and C started a business by investing Rs. 8400, Rs. 15600 and Rs. 13200 respectively. All of them invested for equal period of time. If A’s share of annual profit was Rs. 1960, what was the difference between B’s share in annual profit and C’s share in annual profit?

A. Rs. 560

B. Rs. 520

C. Rs. 540

D. Rs. 480

E. Rs. 600

 

Q. 59 Two pipes A and B can separately fill a cistern in 60 minutes and 75 minutes respectively. There is a third pipe in the bottom of the cistern to empty it. If all the three pipes are simultaneously opened, then the cistern is full in 50 minutes. In how much time the third pipe alone can empty the cistern.

A. 110 minutes

B. 100 minutes

C. 120 minutes

D. 90 minutes

E. 130 minutes

 

Questions: 60 – 64

In the following questions, two equations numbered I and II are given. You have to solve both the equations and – Give answer IF

(1) x < y

(2) x > y

(3) x ≤ y

(4) x ≥ y

(5) x = y or the relationship cannot be established

 

Q. 60 I. 15x² + 26x + 8 = 0

II. 25y² + 15y + 2 =0

A. x < y

B. x > y

C. x ≤ y

D. x ≥ y

E. x = y or the relationship cannot be established

 

Q. 61 I. 6x² – 19x + 15 = 0

II. 5y² – 22y + 24 = 0

A. x < y

B. x > y

C. x ≤ y

D. x ≥ y

E. x = y or the relationship cannot be established

 

Q. 62 I. 4x² – 12x + 5 = 0

II. 4y² – 8y + 3 = 0

A. x < y

B. x > y

C. x ≤ y

D. x ≥ y

E. x = y or the relationship cannot be established

 

Q. 63 I. 10x² + 21x + 8 = 0

II. 5y² + 19y + 18 = 0

A. x < y

B. x > y

C. x ≤ y

D. x ≥ y

E. x = y or the relationship cannot be established

 

Q. 64 I. 6x² – 5x + 1 = 0

II. 12y² – 23y + 10 = 0

A. x < y

B. x > y

C. x ≤ y

D. x ≥ y

E. x = y or the relationship cannot be established

 

Q. 65 The sum of the ages of a father and a son is 45 years. Five years ago, the product of their ages was four times the father’s age at that time. What are the present ages of father and son? (In years)

A. 34 and 11

B. 35 and 10

C. 36 and 9

D. 40 and 5

E. 42 and 12

 

Questions: 66 – 70

Study the following carefully and answer the questions given below :

Seven boxes – R, X, T, U, V, W, and X – are kept one above the other but not necessarily in the same order. Each box contains different elements – Chocolates, Accessories, Ribbons, Balloons, Keys , Pins and Nuts, but not necessarily in the same order. Only three boxes are kept between U and V, The ribbon box is kept immediately above U. Only one box is kept between the ribbon box and the nuts box. The nuts box is kept below the ribbon box. Only two boxes are kept between the nuts box and R. S is kept immediately below X. S is not kept immediately above V. Only two boxes are kept between X and the Balloon box. V does not contain balloons. The chocolate box is kept immediately above W. Only Three boxes are kept between the chocolates box are kept between the chocolates box and the accessories box. W does not contain keys.

 

Q. 66 How many boxes are kept between T and the nuts box?

A. None

B. Two

C. One

D. Three

E. More than three

 

Q. 67 Four of the following five are alike in a certain way and hence form a group. Which of the following does not belong to the group?

A. X – Ribbons

B. S – Pins

C. V – Balloons

D. U – keys

E. R – Chocolates

 

Q. 68 What is the position of S in the given stack of boxes?

A. Second from the top

B. Third from the bottom

C. Fifth from the top

D. Fourth from the top

E. First from the bottom

 

Q. 69 Which of the following boxes contains pins?

A. V

B. S

C. Other than those given as options

D. W

E. X

 

Q. 70 Which of the following boxes is kept immediately below R?

A. The Ribbon Box

B. X

C. W

D. V

E. The Accessories Box

 

Questions: 71 – 75

Study the following carefully and answer the questions given below :

Eight friends, J, K, L, M, S, T, U and V are seated in a straight line, nut not necessarily in the sane order. Some are them are facing north while some face south.

⇒ K sits second from the end of the line. U sits third to the left of K.

⇒ T faces south. V sits fourth to the left of T. T is not an immediate neighbor of U. T does not sit at any of the extreme ends of the line.

⇒ Both the immediate neighbors of J face north. J does not sit at any of the extreme ends of the line.

⇒ J faces a direction opposite to that of V (i.e if V faces north then J faces south and vice – versa).

⇒ L sits second to the left of J.

⇒ Immediate neighbors of K face opposite directions (i.e, if one neighbor faces north then the other faces south and vice versa.)

⇒ Persons sitting at extreme ends face opposite directions (i.e. if one neighbor faces north then the other also faces north and vice versa.)

⇒ Immediate neighbors of U face same direction (i.e. if one neighbour faces north then the other also faces north and vice – versa.)

 

Q. 71 Which of the following statements is true based on the given arrangement?

A. V sits at one of the extreme ends of the line.

B. L faces north

C. None of the given options is true.

D. Only three persons face north

E. S and U face the same directions

 

Q. 72 Who amongst the following faces North?

A. L

B. M

C. J

D. K

E. S

 

Q. 73 Who amongst the following sits third to the left of S?

A. K

B. J

C. V

D. L

E. No one as less than three persons sits to the left of S.

 

Q. 74 How many person(s) sit exactly between T and U?

A. Four

B. Three

C. Two

D. More than four

E. One

 

Q. 75 Who among the following represents the persons sitting at the extreme ends of the line?

A. M, L

B. M, S

C. L, V

D. M, V

E. S, V

 

Questions: 76 – 80

In these questions, relationship between different elements is shown in the statements. The statements are followed by two Conclusions numbered I and II. Study the Conclusions based on the statements and select the appropriate answer.

Give answer (1) if both the Conclusion I and II are true

Give answer (2) if only Conclusion I is true

Give answer (3) if neither Conclusion I nor II is true

Give answer (4) if only Conclusion II is true

Give answer (5) if either Conclusion I or Conclusion II is true

 

Q. 76 Statements :

C < O ≤ Z ≥ Y : N > Z < M

Conclusions :

I. N > Y

II. C < M

A. both the Conclusion I and II are true

B. only Conclusion I is true

C. neither Conclusion I nor II is true

D. only Conclusion II is true

E. either Conclusion I or Conclusion II is true

 

Q. 77 Statements :

P ≤ C < H = Q ≤ T > M ≤ N

Conclusions :

I. P < Q

II. Q > N

A. both the Conclusion I and II are true

B. only Conclusion I is true

C. neither Conclusion I nor II is true

D. only Conclusion II is true

E. either Conclusion I or Conclusion II is true

 

Q. 78 Statements :

P ≤ C < H = Q ≤ T > M ≤ N

Conclusions :

I. H > M

II. T > C

A. both the Conclusion I and II are true

B. only Conclusion I is true

C. neither Conclusion I nor II is true

D. only Conclusion II is true

E. either Conclusion I or Conclusion II is true

 

Q. 79 Statements :

J > K ≥ L < X : D ≥ U > K

Conclusions :

I. L >D

II. X > J

A. both the Conclusion I and II are true

B. only Conclusion I is true

C. neither Conclusion I nor II is true

D. only Conclusion II is true

E. either Conclusion I or Conclusion II is true

 

Q. 80 Statements :

N ≥ O = P ≥ B ≥ R

Conclusions :

I. R < N

II. R = N

A. both the Conclusion I and II are true

B. only Conclusion I is true

C. neither Conclusion I nor II is true

D. only Conclusion II is true

E. either Conclusion I or Conclusion II is true

 

Questions: 81 – 86

Study the following information carefully and answer the questions given below : Seven persons namely O, P, Q, R, S, T and U have to attend a practical but not necessarily in the same order, on seven different months (of the same year) namely, February, March, April, June, August, September and November. Each of them also likes a different subject namely, Chemistry, Biology, Psychology, Hindi, English, Geography and Account but not necessarily in the same order. The one who likes Hindi will attend a practical in a month which has 31 days. Only two persons will attend a practical between the one who likes Hindi and T. Only three persons will attend a practical between T and Q. Only one person will attend a practical between Q and the one who likes Accounts. The one who likes Chemistry will attend a practical in one of the months before the one who likes Accounts. The one who likes Chemistry will attend a practical in the month which has less than 30 days. Only two persons will attend a practical between the one who likes Chemistry and S. Only one person will attend a practical between S and the one who likes Psychology. The one who likes Psychology will attend a practical in one of the months after S. Only two persons will attend a practical between the on who likes Psychology and P. The one who likes Geography will attend a practical immediately before P. Only three persons will attend a practical between P and the one who likes English. Only one person will attend a practical between O and R. O will attend a practical in a month which has 31 days.

 

Q. 81 Who amongst the following likes Biology?

A. P

B. T

C. U

D. R

E. O

 

Q. 82 How many persons will attend a practical after P?

A. Three

B. One

C. Two

D. None

E. More than three

 

Q. 83 Which of the following represents the persons who will attend a practical immediately before and immediately after O?

A. T, P

B. P, Q

C. S, Q

D. S, P

E. U, P

 

Q. 84 As per the given arrangement February is related to T and March is related to P following a certain pattern, which of the following is August related to following the same pattern?

A. U

B. Q

C. R

D. S

E. O

 

Q. 85 Which of the following represents the month in which U will attend a practical?

A. November

B. February

C. April

D. June

E. Cannot be determined

 

Q. 86 Who amongst the following likes English?

A. S

B. T

C. U

D. R

E. Other than those given as options

 

Questions: 87 – 89

Study the following the information carefully and answer the questions given below :

Each of the six buildings, C, D, E, F, G and H has different number of floors. E has the second lowest number of floors. D has more number of floors than C and H but less than F. F does not have the maximum number of floors. H has less number of floors than both G and E. The building having second highest number of floors has 40 floors. C has 28 floors.

 

Q. 87 If the number of floors in buildings E + C is seven more than the number of floors in building F, how many floors are there in building E?

A. 47

B. 19

C. 34

D. 12

E. 15

 

Q. 88 How many floors does building H possibly have?

A. 56

B. 41

C. 30

D. 16

E. 35

 

Q. 89 Which of the following statements is true regarding the number of floors in building G?

A. The difference between number of floors in G and C is less than 12.

B. Only D has more number of floors than G.

C. G has more number of floors than only one building.

D. No other building has less than floors than G.

E. G possibly has 47 floors.

 

Questions: 90 – 95

Study the following information carefully and answer the questions given below:

Seven persons namely D, E, F, G, H, I, and J like seven different monuments namely, Taj Mahal, Red Fort, Charminar, India Gate, Qutub Minar, Victoria Memorial and Sanchi Stupa. Each of them works in either of the three fields viz. Economics, Management and Pharmacy with at least two of them in a field. (Note: None of the information given is necessarily in the same order.) The one who likes Qutub Minar works in field of Management only with G. The one who likes Charminar works with the one who likes Sanchi Stupa, J works with the one who likes India Gate. I neither works with G nor in the field of Pharmacy. J does not like Charminar. D likes Red Fort. D does not work with J. F works with only one person. F does not like Qutub Minar. H works with I. I does not like Charminar. Neither G nor F like Taj Mahal

 

Q. 90 Which of the following represents the combination of that persons who work in the field of Economics?

A. The ones who like Victoria Memorial, Taj Mahal and India Gate

B. The ones who like Charminar, Sanchi Stupa

C. The one who like charminar, Sanchi stupa and victoria memorial.

D. The ones who like Taj Mahal and India Gate

E. The ones who like Sanchi Stupa, Red Fort and Charminar

 

Q. 91 Which of the following is not true?

A. H likes Sanchi Stupa

B. E works only with G

C. All of the given statements are true

D. Both D and H work in the same field

E. Three persons work in the field of Economics

 

Q. 92 Which of the following combinations represent the field in which I works and the monument he likes?

A. Economics – Taj Mahal

B. Pharmacy – India Gate

C. Management – India Gate

D. Economics – Sanchi Stupa

E. Pharmacy – Charminar

 

Q. 93 Four of the following five are alike in a certain way as per the given arrangement and hence form a group. Which of the following does not belong to that group?

A. HI

B. JF

C. GE

D. EJ

E. ID

 

Q. 94 Which of the following monuments does E like?

A. Taj Mahal

B. Qutub Minar

C. Sanchi Stupa

D. Charminar

E. India Gate

 

Q. 95 Who amongst the following like Victoria Memorial?

A. H

B. J

C. G

D. F

E. I

 

Questions: 96 – 97

Point U is 35m to the west of Point W. Point W is 20m to the south of Point H. Point H is 25m to the east of Point S. Shubham is standing at point Z which is 40m to the south of Point S. He starts walking towards east and walks for 30m. He takes a left turn and stops at Point K after walking for 20m.

 

Q. 96 If Shubham walks for 15m towards west from his final position to reach Point V, how much distance will he have to cover in order to reach Point U

A. 30m

B. 15m

C. 35m

D. 25m

E. 20m

 

Q. 97 How far and in which direction is Point W with respect to Point K?

A. 10m towards North

B. 15m towards West

C. 5m towards East

D. 5m towards West

E. 15m towards East

 

Questions: 98 – 100

L is the mother if J. J is the only daughter of R. R is the son of D. D is the wife of K.

K is the father of T. T is the wife of Y.

 

Q. 98 If J is the sister of X, then how is X related to Y?

A. Son

B. Nephew

C. Daughter

D. Niece

E. Daughter-in-law

 

Q. 99 How is L related to T?

A. Niece

B. Sister

C. Daughter-in-law

D. Daughter

E. Sister-in-law

 

Q. 100 How is J related to K?

A. Daughter

B. Granddaughter

C. Daughter-in-law

D. Grandfather

E. Niece

 

 

Answer Sheet
Question 1 2 3 4 5 6 7 8 9 10
Answer C D A B B E A A C D
Question 11 12 13 14 15 16 17 18 19 20
Answer A D D B C C C B D E
Question 21 22 23 24 25 26 27 28 29 30
Answer C A A D B A A A D A
Question 31 32 33 34 35 36 37 38 39 40
Answer B B B C D E E A C B
Question 41 42 43 44 45 46 47 48 49 50
Answer D C C A D C C B D A
Question 51 52 53 54 55 56 57 58 59 60
Answer A B A D D A E A B C
Question 61 62 63 64 65 66 67 68 69 70
Answer A D B A C E B E D A
Question 71 72 73 74 75 76 77 78 79 80
Answer D B C E D A E C B B
Question 81 82 83 84 85 86 87 88 89 90
Answer A E C B B D B D E E
Question 91 92 93 94 95 96 97 98 99 100
Answer A D D B C D D B C B

SBI PO 2015 Prelims Previous Year Paper

SBI PO 2015 Prelims

Section

Questions

Marks

English

30 Questions (1 – 30)

30

Data Interpretation

16 Questions

16

Quantitative Aptitude

19 Questions (42 – 60)

19

Logical Reasoning

35 Questions (66 – 100)

35

Q. 1 Each sentence given below has two blanks. Each blanks indicates that something has been omitted. Choose the word that best fits in the meaning of the sentence as a whole. Realistically, however this disconnect cannot _______ itself very long, sooner rather than _______ the whole will no doubt converge.

A. sustain, later

B. sustained, later

C. submerge, latter

D. harmonize, lately

E. mend, lately

 

Q. 2 Each sentence given below has two blanks. Each blanks indicates that something has been omitted. Choose the word that best fits in the meaning of the sentence as a whole. Digitization will _______ a couple of billion dollars in pay revenues, bring more taxes choice and clean out black money ______ cable.

A. realize, from

B. replenish, off

C. release, from

D. revive, with

E. supply, with

 

Q. 3 Each sentence given below has two blanks. Each blanks indicates that something has been omitted. Choose the word that best fits in the meaning of the sentence as a whole. Public sector infrastructure financing companies could _______ of the ambitious smart cities citing _______ of guaranteed returns on investment.

A. opt for, crisis

B. opt out, lack

C. prefer, excess

D. select, lack

E. opt, cause

 

Q. 4 Each sentence given below has two blanks. Each blanks indicates that something has been omitted. Choose the word that best fits in the meaning of the sentence as a whole. Looks like the oil markets are not only showing ______ between the physical and the financial perspectives from time to time but also ______ between the short term view and the long term realities.

A. disconnect, convergence

B. disconnect, divergence

C. connection, difference

D. supply, demand

E. similarity, contrast

 

Q. 5 Each sentence given below has two blanks. Each blanks indicates that something has been omitted. Choose the word that best fits in the meaning of the sentence as a whole. Top global oil exporter Saudi Arabia ______ its crude production in April to a record high, _______ its flourishing Asian market share.

A. rose, feed

B. risen, collecting

C. raised, lead

D. raised, feeding

E. increased, healing

 

Questions: 6 – 15

Read the following passage carefully and answer the questions given below it.Certain words/phrases are given in CAPITAL to help you locate them while answering some of the questions:

Core competencies and focus are now the mantras of corporate strategists in Western economies. But while managers in the West have (DISMANTLED) many conglomerates assembled in the 1960s and 1970s, the large, diversified business group remains the dominant form of enterprise throughout most emerging markets. Some groups operate as holding companies with full ownership in many enterprises, others are collections of publicly traded companies, but all have some degree of central control. As emerging markets open up to global competition, consultants and foreign investors are increasingly pressuring these groups to (CONFORM TO) Western practice by scaling back the scope of their business activities. The conglomerate is the dinosaur of organizational design, they argue, too unwieldy and slow to compete in today’s fat paced markets. Already a number of executives have decided to break up their groups in order to show that they are focusing on only a few core businesses. There are reasons to worry abut this trend. Focus is good advice in New York or London, but something important gets lost in translation when that advice is given to groups in emerging markets. Western companies take for granted a range of institutions that support their business activities, bu many of these institutions are absent in other regions of the world. Without effective securities regulation and venture capital firms, for example, focused companies may be unable to raise adequate financing; and without strong educational institutions, they will struggle to hire skilled employees. Communicating with customers is difficult when the local infrastructure is poor, and unpredictable government behavior can stymie any operation. Although a focused strategy may enable a company to perform a few activities well, companies in emerging markets must take responsibility for a wide range of functions in order to do business effectively.

In the case of products markets, buyers and sellers usually suffer from a severe (DEARTH)of information for three reasons. First, the communications infrastructure in emerging markets is often under developed. Even as wireless communication spreads throughout the West, vast stretches in countries such as China and India remain without telephones. Power shortages often render the modes of communication that do exist ineffective. The postal service is typically inefficient, slow, or unreliable : and the private sector rarely provides efficient courier services. High rates of illiteracy make it difficult for marketers to communicate effectively with customers. Second, even when information about products does get around, there are no mechanisms to corroborate the claims made by sellers. Independent consumer – in – formation organizations are rare, and government watchdogs agencies are of little use. The few analysts who rate products are generally less sophisticated than their counter – parts in advanced economies. Third, consumers have no redress mechanisms if a product does not deliver on its promise. Law enforcement is often (CAPRICIOUS) and so slow that few who assign any value to time would resort to it. Unlike in advanced markets, there are few extra judicial arbitration mechanisms to which one can appeal. As a result of this lack of information, companies in emerging markets face much higher costs in building credible brands than their counterparts in advanced economies. In turn, established brands wield tremendous power. A conglomerate with a reputation for quality products and services can use its group name to enter new businesses, even if those businesses are completely unrelated to its current lines. Groups also have an advantage when they do try to build up a brand because they can spend the cost of maintaining it across multiple lines of business. Such groups then have a greater incentive not to damage brand quality in any one business because they will pay the price in their other businesses as well.

 

Q. 6 Which of the following sentences is/are correct in the context of the given passage?

I. Consultants and foreign investors argue that the conglomerate is the dinosaur of organizational design too unwieldy and slow to compete in today’s fast – paced markets.

II. Core competencies and focus are now the mantras of corporate strategists in western economies.

III. Due to lack of information required, companies in emerging markets face much higher costs in building credible brands in comparison to their counterparts in advanced economies.

A. Only I

B. Only II and III

C. Only I and III

D. Only I and II

E. All I, II and III

 

Q. 7 What suggestions have been cited by the writer in regard to raising adequate financing and hiring skilled employees?

A. Effective securities regulation

B. Effective securities regulation and venture capital firms

C. Effective securities regulation and venture capital firms and strong educational institutions

D. Both (1) and (3)

E. None of these

 

Q. 8 The writer has cited some hurdles in the case of product markets regarding shortage of information. Which of the following statements(s) in this regard is/are true?

I. Communications infrastructure in emerging markets is often under developed.

II. Postal service is typically inefficient, slow or unreliable.

III. High rates of illiteracy make it difficult for marketers to communicate effectively with customers

A. Only I

B. Only II and III

C. Only I and III

D. Only I and II

E. All I, II and III

 

Q. 9 Which of the following statements is correct in regard to the given passage?

A. The few analysts in emerging markets who rate products are generally less sophisticated than their counterparts in advanced economies.

B. Unlike in advanced markets there are few extra-judicial arbitration mechanisms in emerging markets to which one can appeal.

C. Even as wireless communication spreads throughout the West vast regions of China and India remain without telephones.

D. Unpredictable government behavior can stymie any operation.

E. All are correct

 

Q. 10 Established brands can wield tremendous power in emerging markets because

A. a conglomerate with a reputation for quality products and services can use its group name to enter new businesses.

B. they have much political nexus and strong man power.

C. they have excess of money and customers

D. they have greater incentive to damage brand quality in any one business

E. None of these

 

Q. 11 What should be the most appropriate title of this passage?

A. Hurdles in Emerging market

B. What is Emerging market

C. Lack of information in Emerging markets

D. Advanced Markets Eat Emerging markets

 

Q. 12 Choose the word/group of words which is most similar in meaning to the word/group of words printed in CAPITAL as used in the passage

CONFORM TO

A. comply

B. conflict between

C. confirm

D. confiscate

E. confine to

 

Q. 13 DISMANTLE.

A. take together

B. hold

C. take apart

D. disorder

E. dismount

 

Q. 14 CAPRICIOUS

A. unpredictable

B. predictable

C. changeable

D. captive

E. reasonable

 

Q. 15 DEARTH….

A. scarcity

B. shortage

C. paucity

D. abundance

E. debility

 

Questions: 16 – 20

Rearrange the following six sentences (A), (B), (C), (D), (E) and (F) in the proper sequence to form meaningful paragraph; then answer the questions given below them.

(A) Colony losses last year weren’t as dramatic as the declines associated with Colony Collapse Disorder (CCD), which was first identified in October 2006.

(B) Beekeepers tapped for the survey manage a total of 400,000 colonies, representing about 14.5% of the United States honeybee colonies.

(C) Overall, colony losses during the 12 – month period that ended in April reached 42.1% – the second highest annual loss to date.

(D) Summer colony losses reached 27.4% , exceeding winter losses that came in at 23.7%.

(E) For the first time, beekeepers watched more of their colonies disappear during the summer than in winter.

(F) A new survey outlining honeybee colony losses in the U.S has scientists scratching their heads

 

Q. 16 Which of the following should be the FIRST sentence after rearrangement?

A. A

B. B

C. F

D. E

E. D

 

Q. 17 Which of the following should be the SECOND sentence after the rearrangement?

A. E

B. F

C. A

D. B

E. C

 

Q. 18 Which of the following should be the SIXTH sentence after rearrangement?

A. A

B. B

C. C

D. D

E. E

 

Q. 19 Which of the following should be the FOURTH sentence after rearrangement?

A. A

B. B

C. C

D. D

E. E

 

Q. 20 Which of the following should be the FIFTH sentence after rearrangement?

A. A

B. B

C. C

D. D

E. E

 

Questions: 21 – 25

Read each sentence to find out whether there is any grammatical error or idiomatic error in it. The error, if any, will be in one part of the sentence. The number of that part is the answer. If there is no error, the answer is (5).

 

Q. 21 Profitability of fleet operators(1)/ have improved due to a decline(2)/ in fuel prices during(3)/ the last two months.(4)/ No error(5)

A. 1

B. 2

C. 3

D. 4

E. 5

 

Q. 22 We are a young country, (1)/ a brash country, a forward (2)/ looking country, and (3)/ true history interest us a lot. (4)/ No error(5)

A. 1

B. 2

C. 3

D. 4

E. 5

 

Q. 23 The joint statement included (1)/ just three lines on military (2)/ cooperation, restriction itself for (3)/ exercise and ship visits. (4)/ No error (5)

A. 1

B. 2

C. 3

D. 4

E. 5

 

Q. 24 In a country currently there is (1)/ absolute no shortage in fact(2)/ there is an abundance of pilots holding (3)/ a valid license but unable to find a job.(4)/ No error(5).

A. 1

B. 2

C. 3

D. 4

E. 5

 

Q. 25 WPI might have turned negative primarily(1)/ due to a steep decline in the prices (2)/ of non – food articles(3)/ raising vegetable prices keep food articles firm during this month.(4)/ No error(5)

A. 1

B. 2

C. 3

D. 4

E. 5

 

Questions: 26 – 30

In the following passage there are blanks, each of which has been numbered. These numbers are printed below the passage and against each, five words are suggested, one of which fits the blank appropriate word in each case. There are plenty written about the wealth divide in the U.S. economy. But there is another important divide the one between consumers and corporations. If you look at how U.S. households have been behaving (26) you’d think it was all blue skies. Consumers confidence is at five-year high, thanks to higher stock prices and a (27) in the housing market. Home prices have had their biggest jump since 2005. Consumers, finally feeling more (28) are buying that new car or electronic gadget and bolstering GDP growth a bit. The wealth gap between America’s high income group and everyone else has (29) record high levels since the economic recovery from the great recession of 2007-09, with a clear (30) of increasing wealth for the upper-income families and no wealth growth for the middlelower- income families.

 

Q. 26 Find out the appropriate word for the blank (26)

A. late

B. lately

C. uniformly

D. eager

E. earnestly

 

Q. 27 Find out the appropriate word for the blank (27)

A. discovering

B. growing

C. recovery

D. delivery

E. depletion

 

Q. 28 Find out the appropriate word for the blank (28)

A. flush

B. happy

C. satisfied

D. flung

E. flunk

 

Q. 29 Find out the appropriate word for the blank (29)

A. reach

B. reached

C. delivered

D. targeted

E. subjugated

 

Q. 30 Find out the appropriate word for the blank (30)

A. target

B. projection

C. trajectory

D. tarnation

E. temptation

 

Questions: 31 – 36

Study the following table which details the “Number of students enrolled in 4 different courses of a college during the given years” & answer the questions below:

 

Q. 31 What was the average number of boys studying in all four courses of the college in the year 2010?

A. 493

B. 480

C. 439

D. 468

E. none of these

 

Q. 32 What was the average number of females studying in all four courses of the college in the year 2012?

A. 573

B. 537

C. 437

D. 473

E. none of these

 

Q. 33 By what percentage approximately is the number of boys studying in all four courses of the college in the year 2011 less than that of the girl students studying in all four courses in the same year?

A. 32

B. 30

C. 26

D. 22

E. none of these

 

Q. 34 What is the difference between the number of girls studying number of girls studying in all four courses of the college in the year 2010 and that of boys studying in all four courses of the college in the year 2011?

A. 205

B. 215

C. 305

D. 315

E. none of these

 

Q. 35 What is the respective ratio between the total number of boys in courses B and D together in year 2010 and that of all students in courses A and D in 2012?

A. 3:5

B. 1:3

C. 3:7

D. 4:5

E. none of these

 

Q. 36 A dealer allowed a discount of 25% on the marked price of Rs 12000 on an article and incurred a loss 10%. What discount should be allow on the market price so that he gains Rs. 440 on the article ?

A. 11%

B. 13%

C. 19%

D. 15%

E. none of these

 

Questions: 37 – 41

Study the following table which details the “Number of projects handled by 6 companies during 6 years” & answer the questions below:

 

Q. 37 What is the average number of projects handled by company A during all given years ?

A. 1122/3

B. 1130/3

C. 1129/3

D. 1103/3

E. none of these

 

Q. 38 What is the respective ratio between the total number of projects handled by both companies in the year 2002 and 2003 ?

A. 4:5

B. 5:9

C. 7:9

D. 5:7

E. none of these

 

Q. 39 By what percent is the number of projects handled by company B in the year 2006 more than handled in the year 2002 by the same company ?

A. 84%

B. 86%

C. 72%

D. 76%

E. none of these

 

Q. 40 What is the average number of projects handled by company B during all given years ?

A. 1145/3

B. 1144/3

C. 956/3

D. 1129/3

E. none of these

 

Q. 41 What is the difference between the total number of projects handled by company A and company B in the year 2001, 2003 and 2006 taken together ?

A. 250

B. 230

C. 260

D. 240

E. none of these

 

Q. 42 A man gave 20% of his salary to his only son and to his only daughter. The ratio of amount given to son and daughter is 3:2 respectively. Twice the amount what her gave to his daughter, he invested in LIC. Out of the remaining amount he gave one fourth to his wife. After that he was left with Rs. 16800. Find out the amount invested in LIC ?

A. Rs. 5600

B. Rs. 5400

C. Rs. 5800

D. Rs. 6200

E. none of these

 

Q. 43 In a vessel there is 40 liters mixture of milk and water. There is 15% water in the mixture. The milkman sells 10 liters of mixture to the customer and thereafter adds 12.5 liters of water to the remaining mixture. What is the respective ratio of milk and water in the new mixture ?

A. 2:3

B. 3:2

C. 3:4

D. 4:3

E. none of these

 

Q. 44 A boat covers a distance of 2.75 km upstream in 11 minutes. The ratio between speed of current and the of boat downstream is 1:7 respectively, The boat overs distance between A and B downstream ion 52 minutes. What is the distance between point A and point B ?

A. 19.2 km

B. 17.2 km

C. 18.2 km

D. 16.5 km

E. none of these

 

Q. 45 What will come in the place of the question mark (?) in each of the following number series?
125 128 119 146 65 ?

A. 308

B. 316

C. 298

D. 294

E. none of these264

 

Q. 46 What will come in the place of the question mark (?) in each of the following number series?

8 17 30 47 68 ?

A. 83

B. 93

C. 98

D. 95

E. 96

 

Q. 47 What will come in the place of the question mark (?) in each of the following number series?

24 12 12 18 ? 90

A. 40

B. 38

C. 36

D. 45

E. none of these

 

Q. 48 What will come in the place of the question mark (?) in each of the following number series?

5 16 49 104 ? 280

A. 165

B. 160

C. 171

D. 181

E. 175

 

Q. 49 What will come in the place of the question mark (?) in each of the following number series?

13 19 30 48 75 ?

A. 107

B. 108

C. 116

D. 112

E. 113

 

Q. 50 A and B together can complete a piece of work in 19/7 days while B and C together can complete the same work in 40/3 days. B is 25% more efficient than C. In how many days will A and C together completer the same work ?

A. 45/4

B. 49/4

C. 34/3

D. 37/3

E. none of these

 

Q. 51 The sum of ages of Ria and Abby is 48 years. Today Abby is 4 years older than Shweta. There respective ratio of the present ages of Ria and Shweta is 4:7. What was the Abby’s age two years ago ?

A. 32 years

B. 30 years

C. 28 years

D. 34 years

E. none of these

 

Q. 52 What approximate value will come in place of the question mark (?) in the following questions ? ( You are not expected to calculate the exact value ).

52.02% of 749 + 45% of 419.98 – ? = 225

A. 354

B. 364

C. 370

D. 368

E. none of these

 

Q. 53 What approximate value will come in place of the question mark (?) in the following questions ? ( You are not expected to calculate the exact value ).

349.98 x 19.99 + ?^2 x 180.16 = 11500

A. 3

B. 5

C. 4

D. 9

E. 25

 

Q. 54 What approximate value will come in place of the question mark (?) in the following questions ? ( You are not expected to calculate the exact value ).

( 1800 / sqrt ( ? ) x 29.99 ) / 15.02 = 144

A. 12

B. 25

C. 625

D. 144

E. 169

 

Q. 55 What approximate value will come in place of the question mark (?) in the following questions ? ( You are not expected to calculate the exact value ).

( 52.02^2 – 34.01^2 ) / 17.99 x sqrt ( ? ) = 1720

A. 400

B. 20

C. 25

D. 625

E. ( 340 x 9.98 ) / 6.4001 / 1245.15 = ?

 

Q. 56 What approximate value will come in place of the question mark (?) in the following questions ? ( You are not expected to calculate the exact value ).

( 340 x 9.98 ) / 6.4001 / 1245.15 = ?

A. 1766

B. 1776

C. 1676

D. 1876

E. 1806

 

Q. 57 A, B and C together start a business. The ratio of the investments of A, B and C is 0.125 : 0.75 : 0.25 . After 8 months A adds thrice amount of his earlier investment and C withdraws half of his earlier investment. At the end of the year, they earn a total profit of Rs. 5800. What is B’s share in the profit ?

A. Rs. 3400

B. Rs. 3200

C. Rs. 3600

D. Rs. 3800

E. none of these

 

Q. 58 In a bag there are 4 white balls, 4 red, and 2 green balls. Two balls are drawn at random. What is the probability that at least one ball is green color ?

A. 4/5

B. 3/5

C. 1/5

D. 2/5

E. none of these

 

Q. 59 Equal amounts are invested in two schemes A and B for 6 years and 8 years respectively. Scheme A offers interest at the rate of 12% per annum and scheme B offers interest at the rate of *% per annum . The difference between the interests earned is Rs. 1280. What is the amount invested in each scheme ?

A. Rs. 16000

B. Rs. 16500

C. Rs. 17000

D. Rs. 18000

E. none of these

 

Q. 60 The area of rectangle is equal to the area of a square whose diagonal is 12 sqrt(6) meter. The difference between the length and the breadth of the rectangle of 6 meter. What is the perimeter of the rectangle ? ( in meter )

A. 160 meter

B. 80 meter

C. 82 meter

D. 84 meter

E. none of these

 

Questions: 61 – 65

Directions(61-65): In the following questions two equations numbered I and II are given. You have to solve both the equations

 

Q. 61 I. 3x² + 14x + 15 = 0

II. 6y² + 17y + 12 = 0

A. if x > y

B. if x ≥ y

C. if x < y

D. if x ≤ y

E. if x = y or the relationship cannot be established

 

Q. 62 I. 3x² – 17x + 24 = 0

II. 4y² – 15y + 14 = 0

A. if x > y

B. if x ≥ y

C. if x < y

D. if x ≤ y

E. if x = y or the relationship cannot be established

 

Q. 63 I. 2x² + 11x + 14 = 0

II. 2y² + 17y + 33 = 0

A. if x > y

B. if x ≥ y

C. if x < y

D. if x ≤ y

E. if x = y or the relationship cannot be established

 

Q. 64 I. 3x² + 13x + 12 =0

II. 2y² + 15y + 27 = 0

A. if x > y

B. if x ≥ y

C. if x < y

D. if x ≤ y

E. if x = y or the relationship cannot be established

 

Q. 65 I. x² – 22x + 121 = 0

II. y² = 121

A. if x > y

B. if x ≥ y

C. if x < y

D. if x ≤ y

E. if x = y or the relationship cannot be established

 

Questions: 66 – 70

REASONING

Directions (66-70) : Study the following information carefully and answer the questions given below :

Eight friends — C,D,E,F,N,O,P and Q — are sitting in a straight line facing north but not necessarily in the same order. O is sitting second to the right of P. E is sitting third to the left of Q. F is sitting fourth to the left of P. E does not sit on the extreme end of the line. D is immediate neighbor of neither F nor P. C is not an immediate neighbor of P.

 

Q. 66 Which of the following pairs represents the two persons sitting at the extreme ends of the line?

A. F,C

B. D,F

C. C,Q

D. N,D

E. O,F

 

Q. 67 Which of the following statements is definitely true as per the given arrangement?

A. F and E are immediate neighbors of P

B. C is sitting third to the right of Q

C. There are only three persons between D and E

D. N is sitting to the immediate left of P

E. All the given statements are true

 

Q. 68 Who among the following is sitting third to the right of N?

A. O

B. D

C. C

D. E

E. Q

 

Q. 69 What is the position of C with respect to Q?

A. Fourth to the right

B. Third to the right

C. Fourth to the left

D. Second to the right

 

Q. 70 Four of the following five are alike in a certain way based on the given arrangement and hence they form a group. Which one of the following does not belong to that group?

A. FN

B. PD

C. EQ

D. CP

E. NQ

 

Questions: 71 – 73

Directions (71 – 73 ) : Study the following information carefully and answer the questions given below:

S is the daughter of U. V has only two children – S and Y. Y is married to D. P is the brother of B. V has only two daughters. J is the mother of U. J is married to L. P is married to S. V is the son of T.

 

Q. 71 Who among the following is the sister-in- law of B?

A. Y

B. S

C. U

D. J

E. T

 

Q. 72 Who among the following is the father of U?

A. J

B. T

C. V

D. L

E. None of these

 

Q. 73 How is V related to P?

A. Father-in-law

B. Mother-in-law

C. Father

D. Mother

E. Brother

 

Q. 74 Point A is 30 meters to the east of point B. Point C is 10 metres to the south of Point A. Point D is 15 metres to the west of Point C. Point E is exactly in the middle of the Points D and F. Points D,E and F lie in a straight line. The length of the line DEF is 20 metres. Point F is to the north of point D. Point G is 15 metres to the east of point F. How far and in which direction is Point G from Point A?

A. 10 metres, South

B. 15 metres, North

C. 10 metres, North

D. 15 metres, South

E. 10 metres, East

 

Q. 75 A person starts from his house and moves towards the market. He walks 40 metres towards south and takes a right turn. After walking 30 metres he takes a left turn and walks 20 metres. Finally he takes a left turn and reach the market after walking 30 metres. How far and in which direction is his house from the market?

A. 60 metres, South

B. 60 metres, North

C. 70 metres, North

D. 70 metres, South

E. 90 metres, North

 

Questions: 76 – 80

Directions (76-80) : In each question given below two or three statements followed by two Conlclusions numbered I and II have been given. You have to take the given statements to be true even if they seem to be at variance from the commonly known facts and then decide which of the following Conclusions logically follows from the given statements, disregarding commonly known facts.

Give answer (1) if only Conclusion I follows

Give answer (2) if only Conclusion II follows

Give answer (3) if either Conclusion I or Conclusion II follows

Give answer (4) if neither Conclusion I nor Conclusion II follows

Give answer (5) if both Conclusion I and Conclusion II follow

 

Q. 76 Statements

All magazines are journals.

Some journals are periodicals.

All periodicals are bulletins.

Conclusions

I. Some periodicals are definitely not journals.

II. All periodicals being magazines is a possibility.

A. if only Conclusion I follows

B. if only Conclusion II follows

C. if either Conclusion I or Conclusion II follows

D. if neither Conclusion I nor Conclusion II follows

E. if both Conclusion I and Conclusion II follow

 

Q. 77 Statements

All magazines are journals.

Some journals are periodicals.

All periodicals are bulletins.

Conclusions:

I. At least some bulletins are journals.

II. No bulletin is a magazine.

A. if only Conclusion I follows

B. if only Conclusion II follows

C. if either Conclusion I or Conclusion II follows

D. if neither Conclusion I nor Conclusion II follows

E. Give answer (5) if both Conclusion I and Conclusion II follow

 

Q. 78 All turns are loops.

No loop is a bend.

Some bends are curves.

Conclusions:

I. At least some curves are loops.

II. No bend is a turn.

A. if only Conclusion I follows

B. if only Conclusion II follows

C. if either Conclusion I or Conclusion II follows

D. if neither Conclusion I nor Conclusion II follows

E. if both Conclusion I and Conclusion II follow

 

Q. 79 Statements:

No country is a village.

All villages are districts.

Conclusions:

I. All countries are districts.

II. All dristricts are villages.

A. if only Conclusion I follows

B. if only Conclusion II follows

C. if either Conclusion I or Conclusion II follows

D. if neither Conclusion I nor Conclusion II follows

E. if both Conclusion I and Conclusion II follow

 

Q. 80 Statements

All progress are growth.

All developments are growth.

No growth is an evolution.

Conclusions:

I. All developments being progress is a possibility.

II. No evolution is a progress.

A. Give answer (1) if only Conclusion I follows

B. Give answer (2) if only Conclusion II follows

C. Give answer (3) if either Conclusion I or Conclusion II follows

D. Give answer (4) if neither Conclusion I nor Conclusion II follows

E. Give answer (5) if both Conclusion I and Conclusion II follow

 

Questions: 81 – 85

Directions (81-85) : In each of the following questions, relationship between different elements is shown in the statements. The statements are followed by two Conclusions numbered I and II. Study the conlcusions based on the given statements and select the appropriate answer :

Give answer (1) if only Conclusion I true

Give answer (2) if only Conclusion II true

Give answer (3) if either Conclusion I or Conclusion II true

Give answer (4) if neither Conclusion I nor Conclusion II true

Give answer (5) if both Conclusion I and Conclusion II true

 

Q. 81 Statements

Y ≤ K < D = S ; D < V < O; G ≥ D < Q

Conclusions:

I. G > V

II. Y < Q

A. if only Conclusion I true

B. if only Conclusion II true

C. if either Conclusion I or Conclusion II true

D. if neither Conclusion I nor Conclusion II true

E. if both Conclusion I and Conclusion II true

 

Q. 82 Statements

Y ≤ K < D = S ; D < V < O; G ≥ D < Q

Conclusions :

I. K < O

II. G = V

A. if only Conclusion I true

B. if only Conclusion II true

C. if either Conclusion I or Conclusion II true

D. if neither Conclusion I nor Conclusion II true

E. if both Conclusion I and Conclusion II true

 

Q. 83 Statements :

D < L ≤ F = N ; L =A

Conclusions:

I. N > D

II. A ≤ F

A. if only Conclusion I true

B. if only Conclusion II true

C. if either Conclusion I or Conclusion II true

D. if neither Conclusion I nor Conclusion II true

E. if both Conclusion I and Conclusion II true

 

Q. 84 Statements

B > Z = R ≥ M < J ≤ H; J > P ; K < Z

Conclusions

I. H < P

II. B > M

A. if only Conclusion I true

B. if only Conclusion II true

C. if either Conclusion I or Conclusion II true

D. if neither Conclusion I nor Conclusion II true

E. if both Conclusion I and Conclusion II true

 

Q. 85 Statements

B > Z = R ≥ M < J ≤ H; J > P ; K < Z

Conclusions

I. K < J

II. R ≥ H

A. if only Conclusion I true

B. if only Conclusion II true

C. if either Conclusion I or Conclusion II true

D. if neither Conclusion I nor Conclusion II true

E. if both Conclusion I and Conclusion II true

 

Questions: 86 – 90

Directions (86-90) : Study the following information carefully and answer the questions given below :

In a certain code language,

‘ good time to buy ‘ is written as ‘ sy bo nj kw ‘.

‘invest money and time’ is written as ‘sy ta ge mr’.

‘only work and money’ is written as ‘ta fp mr ux’.

‘buy good stuff only’ is written as ‘kw bo rd fp’.

 

Q. 86 What is the code for “to” in the given code language ?

A. ge

B. kw

C. nj

D. sy

E. bo

 

Q. 87 What is the code for ” buy good” in the given code language?

A. bo kw

B. kw nj

C. rd bo

D. rd nj

E. Cannot be determined

 

Q. 88 What is the code for ” only time and money ” in the given code language ?

A. sy bo ux fp

B. fp ta rd kw

C. ge fp ta bo

D. mr ta sy fp

E. bo nj ta ge

 

Q. 89 what is the code for “stuff” in the given code language?

A. fp

B. rd

C. kw

D. bo

E. Either ‘bo’ or ‘rd’

 

Q. 90 What is the code for “invest time to work ” in the given code language?

A. sy bo mr fp

B. ta nj kw rd

C. ta fp ux nj

D. mr sy bo ta

E. ux ge nj sy

 

Questions: 91 – 95

Directions (91-95) : Study the following information carefully and answer the questions given below:

Eight persons – M,N,O,P,Q,R,S And T — are sitting around a circular table at equal distance between each other,but not necessarily in the same order. Some of them are facing the centre while some others are facing outside (i.e. in a direction opposite to the centre)

Note: Facing the same direction means if one faces the centre then the other also faces the centre and vice – versa. Facing opposite directions means if one person faces the centre then the other person faces outside and vice- versa. R is sitting second to the right of Q. Only three persons are sitting between R and S. T is sitting second to the right of R . T faces the centre. R and S face opposite directions.P and S face opposite Ddirections. N is sitting second to the left of P. P is not an immediate neighbour of Q. Only one person is sitting between P and O. O is not a n immediate neighbour of Q. M is sitting third to the left of T. The immediate neighbors of T face opposite directions . M and R face opposite directions. N faces the same direction as that of O.

 

Q. 91 Which of the following statements is true regarding T according to the given seating arrangement?

A. T is sitting second to the left of S.

B. T is sitting exactly between O and P.

C. T is sitting just opposite to N.

D. There are four persons between T and Q.

E. T faces the opposite direction as that of M.

 

Q. 92 How many persons in the given seating arrangement face outside?

A. Three

B. Four

C. Five

D. Six

E. Two

 

Q. 93 Four of the following five are alike in a certain way based on the given seating arrangement and hence they form a group. Which is the one that does not belong to that group?

A. O

B. S

C. M

D. Q

E. N

 

Q. 94 Who among the following sits exactly between S and Q when counted from the left of S?

A. None

B. O

C. M

D. N

E. T

 

Q. 95 What is the position of M with respect to R?

A. Other than those given as options

B. third to the right

C. Second to the left

D. Second to the right

E. Third to the left

 

Questions: 96 – 100

Directions (96-100) : Study the following information carefully and answer the questions given below:

Seven persons — M,N,O,P,Q,R And S — live on separate floors of a seven – sotreyed building, but not necessarily in the same order. The ground floor of the building is numbered 1, the floor above it 2 and so on until the topmost floor is numbered 7. Each person likes different cartoon characters,viz, Chipmunk, Flinstone , Jetson , Popeye , Scooby Doo, Simpson and Tweety , but not necessarily in the same order. The person who likes Popeye lives on floor numbered 4. Only two persons live between P and the one who likes Popeye . M does not live on the lowermost floor below the one who likes Popeye. S lives on an even numbered floor but neither immediately above nor immediately below the floor of M. Only two persons live between M and the person who likes Tweety. Only one person lives between N and R. R lives on an even numbered floor and does not like Popeye. Only three persons live between the persons who like Chipmunk and Jetson respectively. The person who likes Chipmunk does not live on the topmost floor. O does not like Chipmunk or Jetson. The person who likes Scooby Doo lives on the floor of the person who likes Simpson.

 

Q. 96 How many persons live between the floors on which S and P live?

A. Three

B. Two

C. Four

D. Five

E. No one

 

Q. 97 Which of the following statements is / are true according to the given information.

A. Q lives on floor numbered 5 and he doesnot like Popeye.

B. M likes Scooby Doo and he does not live on floor numbered 4.

C. O likes Flinstione and he lives on the topmost floor.

D. Only two persons live between the floors of Q and R.

E. All the statements are true.

 

Q. 98 Who among the following lives on the floor immediately above the floor of M?

A. N

B. R

C. S

D. O

E. No One

 

Q. 99 Who among the following lives exactly between the floors on which S and N live?

A. P

B. R

C. M

D. Q

E. No One

 

Q. 100 Who among the following does like cartoon character Jetson?

A. R

B. P

C. N

D. Q

E. S

 

 

Answer Sheet
Question 1 2 3 4 5 6 7 8 9 10
Answer A C B B D E C E E A
Question 11 12 13 14 15 16 17 18 19 20
Answer A A C B E C A A C B
Question 21 22 23 24 25 26 27 28 29 30
Answer B D B B D B C A B C
Question 31 32 33 34 35 36 37 38 39 40
Answer A B C D A B B C D A
Question 41 42 43 44 45 46 47 48 49 50
Answer B A B C A B C D E A
Question 51 52 53 54 55 56 57 58 59 60
Answer B A B C A B C D A D
Question 61 62 63 64 65 66 67 68 69 70
Answer A B B D D B D A C E
Question 71 72 73 74 75 76 77 78 79 80
Answer B D A C B B A B D E
Question 81 82 83 84 85 86 87 88 89 90
Answer B A E B D C A D B E
Question 91 92 93 94 95 96 97 98 99 100
Answer B A D C E C E A D B

SBI PO 2014 Prelims Previous Year Paper

SBI PO 2014 Prelims

Section

Question

Marks

Logical Reasoning

50 Question (1 – 50)

50

Data Interpretation

50 Question (51 – 100)

50

Questions: 1 – 2

Directions (1-2) :

Study the following information carefully and answer the questions given below. There are five statues – L, M, N, O and P – each of them having different height. Statue L is smaller than only statue M. Statue O is smaller than statue N. Statue O is longer than statue P. The height of the tallest statue is 20 feet. The height of the second smallest statue is 11 feet.

 

Q. 1 What will be the height of statue P?

A. 13 feet

B. 15 feet

C. 9 feet

D. 12 feet

E. 14 feet

 

Q. 2 What will be the height of the third tallest statue?

A. 13 feet

B. 10 feet

C. 19 feet

D. 9 feet

E. 11 feet

 

Q. 3 If the expressions S=T>O≥R and P≤O

A. T>O

B. R>S

C. Z>R

D. P>T

E. P<Z

 

Questions: 4 – 10

Directions (4-10) :

Study the following information carefully and answer the questions given below. Eight persons – H, I, J, K, L, M, N and O – are standing in a straight line at equidistant. Some of them are facing north while others are facing south. M is standing third to the right to H. M is standing at one of the extreme ends. L is standing third to the left of H. The immediate neighbours of J face north. N is not an immediate neighbour of H. The persons standing at the extreme ends face the same direction (Both are facing either north or south). The immediate neighbours of H face just opposite direction as that of M. The immediate neighbours of O face opposite directions with respect to each other. One of the immediate neighbours of L is K who is facing north. I is standing between J and M. Not more than four persons are facing north.

 

Q. 4 Who among the following is third to the left of N?

A. K

B. J

C. H

D. I

E. O

 

Q. 5 The immediate neighbours of L are :

A. M and N

B. N and O

C. K and N

D. N and H

E. J and H

 

Q. 6 How many persons are standing exactly between I and O?

A. Three

B. Four

C. One

D. Two

E. None

 

Q. 7 Who among the following is to the immediate left of H?

A. O

B. J

C. I

D. L

E. K

 

Q. 8 Four of the following five are alike in a certain way based on the above arrangement and hence form a group. Which one of the following does not belong to that group?

A. N

B. L

C. O

D. I

E. K

 

Q. 9 Who among the following is exactly between L and J?

A. N

B. O

C. H

D. I

E. None

 

Q. 10 Who among the following is fourth to the right of J?

A. N

B. I

C. H

D. L

E. K

 

Questions: 11 – 15

Direction (11-15) :

In these questions, relationship between different elements is shown in the statements.

The statements are followed by two conclusions. Select the answer based on the conclusions.

 

Q. 11 Statements :

B>C = D≥X; E≤X; Z≥D

Conclusions :

I. B>E

II. Z≥B

A. Conclusion I is true

B. Conclusion II is true

C. Either conclusion I or II is true

D. Neither conclusion I nor II is true

E. Both conclusions I and II are true

 

Q. 12 Statements :

E>F≥GConclusions :

I. G≤E

II. J≥F

A. Conclusion I is true

B. Conclusion II is true

C. Either conclusion I or II is true

D. Neither conclusion I nor II is true

E. Both conclusions I and II are true

 

Q. 13 Statements :

K≤LN≥O;T>M≤P

Conclusions :

I. T>K

II. P>O

A. Conclusion I is true

B. Conclusion II is true

C. Either conclusion I or II is true

D. Neither conclusion I nor II is true

E. Both conclusions I and II are true

 

Q. 14 Statements :

B>O = K≥L; D>K≥S

Conclusions :

I. OII. S≤L

A. Conclusion I is true

B. Conclusion II is true

C. Either conclusion I or II is true

D. Neither conclusion I nor II is true

E. Both conclusions I and II are true

 

Q. 15 Statements :

B>O = K≥L; D>K≥S

Conclusions :

I. L>D

II. B>S

A. Conclusion I is true

B. Conclusion II is true

C. Either conclusion I or II is true

D. Neither conclusion I nor II is true

E. Both conclusions I and II are true

 

Questions: 16 – 20

Directions (16-20) :

Each of the questions below consists of a question and two statements numbered I and II given below it. You have to decide whether the data provided in the statements are sufficient to answer the question.

 

Q. 16 What is the code of ‘shine’ in a certain code language?

I. In that code language ‘shine was peeled off’ is written as ‘&#@9’ and ‘no paint but shine’ is written as ‘75#8’.

II. In that code language ‘try the new shine’ is written as ‘13#0’ and ‘we try the new’ is written as ’6013’.

A. Statement I alone is sufficient to answer the question, while the data in statement II alone are not sufficient to answer the question

B. Statement II alone is sufficient to answer the question, while the data in statement I alone is not sufficient to answer the question.

C. Either statement I or II alone is sufficient to answer the question

D. Both the statements I and II together are not sufficient to answer the question

E. Both the statements I and II together are necessary to answer the questions

 

Q. 17 How C is related to H?

I. N is son of H. J is mother of Z. N and Z are cousins. C is husband of J.

II. L is father of C. A is mother of D. L is married to A. H is wife of D. J is wife of C.

A. Statement I alone is sufficient to answer the question, while the data in statement II alone are not sufficient to answer the question

B. Statement II alone is sufficient to answer the question, while the data in statement I alone is not sufficient to answer the question.

C. Either statement I or II alone is sufficient to answer the question

D. Both the statements I and II together are not sufficient to answer the question

E. Both the statements I and II together are necessary to answer the questions

 

Q. 18 In which direction point ‘A’ is located with respect to point ‘B’?

I. A man starts walking from point ‘A’ towards east after walking 3 metres reaches point ‘N’, he turns right and walks 7 metres to reach point ‘M’. Then he turns right and walks 6 metres to reach point ‘O’. He again turns right and walks 7 metres to reach point ‘P’. He then turns left and walks 2 metres to reach point ‘B’.

II. A man starts walking from point ‘A’ towards east and after walking 3 metres reaches

point ‘N’. From point ‘N’ he walks 7 metres towards south and reaches point ‘M’. From point ‘M’ he walks 6 metres towards west and reaches point ‘O’. From point ‘O’ he walks 7 metres towards north and reaches point ‘P’. From point ‘P’ he walks towards west and reaches point ‘B’. The distance between points A and B is 8 metres.

A. Statement I alone is sufficient to answer the question, while the data in statement II alone are not sufficient to answer the question

B. Statement II alone is sufficient to answer the question, while the data in statement I alone is not sufficient to answer the question.

C. Either statement I or II alone is sufficient to answer the question

D. Both the statements I and II together are not sufficient to answer the question

E. Both the statements I and II together are necessary to answer the questions

 

Q. 19 How many students scored more than B in a class of 25 students?

I. More than four but less than ten students scored more than that of B. B’s rank is an odd number. Seventeen students scored less than D.

II. The rank of C is 16th from the last. B got more marks than C. Only two students are there between B and C.

A. Statement I alone is sufficient to answer the question, while the data in statement II alone are not sufficient to answer the question

B. Statement II alone is sufficient to answer the question, while the data in statement I alone is not sufficient to answer the question.

C. Either statement I or II alone is sufficient to answer the question

D. Both the statements I and II together are not sufficient to answer the question

E. Both the statements I and II together are necessary to answer the questions

 

Q. 20 Five persons – A, B, C, D and E – are sitting around a circular table. Some of them are facing towards centre while others are facing outside.

Who is sitting second to the left of A?

I. B is facing outside. C is to the immediate left of B. D is the second to the right of C. A is to the immediate left of E.

II. D is to the immediate right of A. Both D and A are facing towards the centre. D and B are immediate neighbours of each other.

A. Statement I alone is sufficient to answer the question, while the data in statement II alone are not sufficient to answer the question

B. Statement II alone is sufficient to answer the question, while the data in statement I alone is not sufficient to answer the question.

C. Either statement I or II alone is sufficient to answer the question

D. Both the statements I and II together are not sufficient to answer the question

E. Both the statements I and II together are necessary to answer the questions

 

Questions: 21 – 27

Directions (21-27) :

Study the following information carefully and answer the questions given below. A word and number arrangement machine when given an input rearranges them following a particular rule in each step. The following is an illustration of input and various steps of rearrangement. (All the numbers are two digit numbers). Input : sweet 46 nice 36 friend 26 help 96 bright 76 kind 66

Step I : sweet 46 nice 36 friend 26 help bright 76 kind 66 96

Step II : sweet nice 46 36 friend 26 help bright kind 66 76 96

Step III : sweet nice kind 46 36 friend 26 help bright 66 76 96

Step IV : sweet nice kind help 36 friend 26 bright 46 66 76 96

Step V : sweet nice kind help friend 26 bright 36 46 66 76 96

Step VI : sweet nice kind help friend bright 26 36 46 66 76 96

And step VI is the last step of the rearrangement as the desired arrangement is obtained. As per rules followed in the above steps, find out in each of the questions the appropriate step for the given input. Input : arrow 98 paint 58 lamb 38 each 78 great 18 most 48 rent 88

 

Q. 21 Which word/number would be fifth to the left of the sixth element from the right in the step V?

A. Great

B. Arrow

C. Lamb

D. 38

E. 48

 

Q. 22 Which of the following represents the position of “58” in the step IV?

A. Eighth from left

B. Third from right

C. Ninth from left

D. Eleventh from left

E. Fifth from right

 

Q. 23 How many elements (words/numbers) are there between “most” and “78” as they appear in the step VI?

A. Eight

B. Seven

C. Nine

D. Five

E. Four

 

Q. 24 Which step number is the following output?

Rent paint most arrow 58 lamb 38 each great 18 48 78 88 98

A. There is no such step

B. Step II

C. Step V

D. Step VI

E. Step III

 

Q. 25 Which element (word/number) would be at the eleventh position from the right in the step III?

A. Lamb

B. Arrow

C. 58

D. 38

E. Each

 

Q. 26 Which element (word/number) would be at the sixth position from the left in the step VI?

A. 18

B. Arrow

C. Great

D. Each

E. 38

 

Q. 27 At which of the following positions “great” would appear from the left in the step V?

A. Fifth

B. Sixth

C. Fourth

D. Second

E. Third

 

Questions: 28 – 34

Directions (28-34) :

Study the following information carefully and answer the questions given below : Eight friends – R, S, T, U, V, W, X and Y – are sitting around a circular table facing the centre, but not necessarily in the same order. Each of them studies in the different standards viz, standard I to standard VIII, but not necessarily in the same order. T is second to the right of the person who studies in standard VIII. Only one person sits between T and the person who studies in standard V. X is sitting third to the left of the person who studies in standard VIII. The person studying in standard VIII is not an immediate neighbour of the person studying in standard VII. T does not study in standard VIII. The person studying in standard VI is to the left of U. U does not study in standard V or standard VIII. The person studying in standard VI and VII are immediate neighbours of each other. One of the immediate neighbours of the person studying in standard VIII, studies in standard II. S and Y are immediate neighbours to each other. There is one person between S and R. V does not study in standard I. R studies in standard V. Y does not study in standard II. W is sitting between the persons who study in standard VII and standard IV. X is second to the right of the person studying in standard I.

 

Q. 28 Who among the following is sitting third to the right of T?

A. The person studying in standard II

B. The person studying in standard V

C. Y

D. The person studying in standard I

 

Q. 29 Who among the following is sitting just opposite to R?

A. The person studying in standard VI

B. W

C. The person studying in standard V

D. S

E. The person studying in standard VII

 

Q. 30 Which of the following pairs represents the immediate neighbours of V?

A. R and the person studying in standard VIII

B. S and the person studying in standard III

C. The person studying in standard V and Y

D. Cannot be determined

E. None of these

 

Q. 31 What is the position of W with respect to S?

A. Fourth to the right

B. Fourth to the left

C. Third to the right

D. Fifth to the right

E. Sixth to the left

 

Q. 32 Starting from R, if all the persons are made to sit in the alphabetical order in clockwise direction, the positions of how many (excluding R) will remain unchanged?

A. Two

B. Three

C. Four

D. One

E. None

 

Q. 33 Who among the following is sitting fourth to the left of the person who studies in standard II?

A. U

B. W

C. X

D. S

E. Cannot be determined

 

Q. 34 Who among the following is sitting exactly between the person who studies in standard III and W?

A. T

B. U

C. R

D. V

E. Y

 

Questions: 35 – 40

Directions (35-40) :

Study the following information carefully and answer the questions given below. A college planned to conduct an education fair. The college asked foreign universities to participate in the education fair so that students could know about the foreign universities. Even foreign universities can tell which course is good in which university. The courses offered by these universities are Social Science, Psychology, Medicine, Commerce, Astronomy, Computer Science and Architecture. The fair will start on Monday and will end on Sunday of the same week. Social Science stall will be set up immediately before the Medicine stall. Medicine stall will not be set up on Tuesday or Thursday. There will be a gap of one day between the stalls of Astronomy and Medicine. Architecture stall will be set up immediately after Astronomy stall. Computer Science stall will be set up on Monday.

 

Q. 35 Which of the following stalls will be set up on Thursday?

A. Astronomy

B. Psychology

C. Medicine

D. Architecture

E. None of these

 

Q. 36 Which of the following stalls will be set up immediately after the Computer Science stall?

A. Medicine

B. Commerce

C. Psychology

D. Social Science

E. None of these

 

Q. 37 On which of the following days of the week the stall on architecture will be set up?

A. Saturday

B. Tuesday

C. Thursday

D. Friday

E. None of these

 

Q. 38 Which of the following pairs of stalls will be set up between the stalls on Computer Science and Psychology?

A. Social Science and Astronomy

B. Medicine and Astronomy

C. Social Science and Medicine

D. Medicine and Commerce

E. None of these

 

Q. 39 Which of the following stalls will be set up on Friday?

A. Commerce

B. Medicine

C. Psychology

D. Architecture

E. None of these

 

Q. 40 Which of the following stalls will be set up on Wednesday?

A. Medicine

B. Architecture

C. Psychology

D. Social Science

E. None of these

 

Questions: 41 – 45

Directions (41-45) :

Below in each question are given two statements (A) and (B). These statements may be either independent causes or may be effects of independent causes or a common cause. One of these statements may be the effect of the other statement. Read both the statements and decide which of the options correctly depicts the relationship between the two statements.

 

Q. 41 (A) A substantial increase in unhealthy competition has been observed among the

Students.

(B) A rise of 23% is reported every year in the cases of suicides after declaration of grade 10th and 12th examination results.

A. (A) is the cause and (B) is its effect

B. (B) is the cause and (A) is its effect

C. Both statement (A) and (B) are independent causes

D. Both (A) and (B) are effects of independent causes

E. Both (A) and (B) are effects of some common cause

 

Q. 42 (A) The glaciers at the poles of the earth are melting at a fast rate.

(B) In recent times there has been a substantial increase in the incidents of earthquakes

and volcanic eruptions.

A. (A) is the cause and (B) is its effect

B. (B) is the cause and (A) is its effect

C. Both statement (A) and (B) are independent causes

D. Both (A) and (B) are effects of independent causes

E. Both (A) and (B) are effects of some common cause

 

Q. 43 (A) Most of the shopkeepers in the locality closed their shops for the second continuous day.

(B) Two groups of people living in the locality have been fighting with each other with bricks and stones forcing people to stay indoors

A. (A) is the cause and (B) is its effect

B. (B) is the cause and (A) is its effect

C. Both statement (A) and (B) are independent causes

D. Both (A) and (B) are effects of independent causes

E. Both (A) and (B) are effects of some common cause

 

Q. 44 (A) The government has decided to increase the prices of LPG gas cylinders with immediate effect.

(B) The government has decided to increase the prices of kerosene with immediate effect.

A. (A) is the cause and (B) is its effect

B. (B) is the cause and (A) is its effect

C. Both statement (A) and (B) are independent causes

D. Both (A) and (B) are effects of independent causes

E. Both (A) and (B) are effects of some common cause

 

Q. 45 (A) The country ‘X’ has banned the import of fruit.

(B) The intake of that fruit causes disease.

A. (A) is the cause and (B) is its effect

B. (B) is the cause and (A) is its effect

C. Both statement (A) and (B) are independent causes

D. Both (A) and (B) are effects of independent causes

E. Both (A) and (B) are effects of some common cause

 

Questions: 46 – 50

Directions (46-50) :

In each questions below are given three statements followed by two conclusions I and II. You have to assume everything in the statements to be true even if they seem to be at variance from commonly known facts and then decide which of the conclusions logically follows from the information given in the statements.

 

Q. 46 Statements :

No star is a cone.

Some cones are triangles.

All kites are stars.

Conclusion :

I. All stars are kites

II. At least some triangles are stars.

A. Only conclusion I follows

B. Only conclusion II follows

C. Either conclusion I or conclusion II follows

D. There is possibility that some stars are triangles

E. Neither conclusion I nor conclusion II follows

 

Q. 47 Statements :

All drums are banjos.

Some drums are guitars.

No banjo is a flute.

Conclusions :

I. Some guitar are flutes

II. No guitar is flute.

A. Only conclusion I follows

B. Only conclusion II follows

C. There is possibility that some flutes are banjos

D. Either conclusion I or conclusion II follows

E. Neither conclusion I nor conclusion II follows

 

Q. 48 Statements :

Some pins are needles.

All needles are swords.

Some swords are knives.

Conclusions :

I. All swords being pins is a possibility.

II. No needle being knife is a possibility.

A. Only conclusion I follows

B. Only conclusion II follows

C. Either conclusion I or conclusion II follows

D. Neither conclusion I nor conclusion II follows

E. There is possibility that some pins are knives

Q. 49 Statements :

Some schemes are offers.

Some offers are discount.

No discount is a loan.

Conclusions :

I. Those offers which are discounts can never be loans.

II. Some loans are definitely schemes.

A. Only conclusion I follows

B. There is possibility that all schemes are loans

C. Only conclusion II follows

D. Either conclusion I or conclusion II follows

E. Neither conclusion I nor conclusion II follows

 

Q. 50 Statements :

No car is hotel.

All listed are hotels.

No lodge is house.

Conclusions :

I. Some houses are not lodges

II. No lodge is car

A. There is possibility that all houses are cars

B. Only conclusion I follows

C. Only conclusion II follows

D. Neither conclusion I nor conclusion II follows

E. Both conclusion I and conclusion II follows

 

Questions: 51 – 57

Directions (51-57) :

Study the following information carefully and answer the questions.

For a room, the rate of painting is Rs. 3200 per square metre. The rate of carpeting per square metre is 120% of that of tiling. The cost of decorating the room is 14 times to that of carpeting the floor. The cost of electrification is 75% of that of carpeting the floor. The rate of tiling on the floor is 125% of that of painting. The dimensions of the room are 6 m x 6 m x 5 m.

 

Q. 51 What is the ratio of the cost of painting the four walls of the room and that of decoration?

A. 10:63

B. 10:61

C. 10:21

D. 21:10

E. None of these

 

Q. 52 What will be the total cost of decorating the room and tiling the floor when the four walls have also been tiled to a height of 0.25 metre?

A. Rs. 5287200

B. Rs. 2587200

C. Rs. 2588200

D. Rs. 2577200

E. None of these

 

Q. 53 What will be the total cost of painting, carpeting, decoration and electrification of the room if the dimensions of the room be 21 m x 42 m x 27 m?

A. Rs. 30888000

B. Rs. 8388000

C. Rs. 80388000

D. Rs. 40888000

E. None of these

 

Q. 54 What will be the cost of tiling the floor of the room if the rate of tiling be increased by 75% and the dimensions of the room be 51 m x 59 m x 84 m?

A. Rs. 21163000

B. Rs. 1263000

C. Rs. 2163000

D. Rs. 21063000

E. None of these

 

Q. 55 If the length of the room be increased by 20%, breadth by 32% and height by 12%, then what will be the total cost of painting of the four walls of the room and tiling the floor?

A. Rs. 76996.80

B. Rs. 67996.80

C. Rs. 67796.80

D. Rs. 76796.80

E. None of these

 

Q. 56 What will be the total cost of carpeting, decoration, electrification and tiling the floor if the rate of painting be doubled?

A. Rs. 5771200

B. Rs. 5731200

C. Rs. 5371200

D. Rs. 7571200

E. None of these

 

Q. 57 What will be the cost of decoration of the room if it’s cost be increased by 25%?

A. Rs. 60480

B. Rs. 604800

C. Rs. 6048000

D. Rs. 6448000

E. None of these

 

Questions: 58 – 63

Directions (58-63) :

In the following multiple bar diagram, the data regarding cost of production and sales revenue of the company XYZ in the given years have been given. Study the bar diagram carefully and wander the questions.

 

Q. 58 By what percent is the amount received from the sales in the year 2005 of the company, more than the expenditure on production in the year 2008?

A. 45%

B. 40%

C. 48%

D. 49%

E. 50%

 

Q. 59 In how many years the cost of production is more than the average cost of production of the given years?

A. 1

B. 2

C. 3

D. 4

E. None of these

 

Q. 60 In how many years the sales revenue is less than the average sales revenue of the given years?

A. 1

B. 3

C. 2

D. 4

E. None of these

 

Q. 61 If the cost of production in 2004 and 2007 be increased by 25% and 30% respectively, then by what percent will the total cost in these both years be more than the sales revenue of the year 2008?

A. 62.7%

B. 65.7%

C. 67.7%

D. 68.5%

E. None of these

 

Q. 62 In which year the company had maximum profit percent?

A. 2008

B. 2007

C. 2006

D. 2005

E. 2004

 

Q. 63 If the sales revenues in the year 2005, 2006 and 2007 increased by 20%, 25% and 30% respectively and the costs of the production in the years 2007, 2008 and 2009 increased by 20%, 25% and 35% respectively what will be the difference between average sales revenue and average body of production?

A. Rs. 185.59 thousands

B. Rs. 188.59 thousands

C. Rs. 174.59 thousands

D. Rs. 200.59 thousands

E. None of these

 

Questions: 64 – 69

Directions (64-69) :

In the following multiple graphs, production of wheat (in quintals) by the states Bihar, Madhya Pradesh and Punjab have been given. Study the following graphs carefully to answer the questions.

 

Q. 64 If the production of wheat by Madhya Pradesh in the years 2003, 2004, 2005 and 2007 increase by 30%, 40%, 45% and 40% respectively, what will be the overall percentage increase in the production of wheat in the state in the given years?

A. 22%

B. 25%

C. 35%

D. 16%

E. 19%

 

Q. 65 What was the average production of wheat by all three states in the year 2005?(in quintals)

A. 2866+1/3

B. 2866+2/3

C. 2688+2/3

D. 2688+1/3

E. None of these

 

Q. 66 In the given years, what is the average production of wheat in Bihar? (in quintals)

A. 3068

B. 3076

C. 3086

D. 3088

E. None of these

 

Q. 67 If the productions of wheat in Bihar in the years 2001, 2002, 2003 and 2004 increase by 20%, 25%, 28% and 35% respectively; what will be the percentage increase in the average productions of the state for the given years?

A. 35.7%

B. 38.7%

C. 40.7%

D. 42 5%

E. None of these

 

Q. 68 By what percent is the total production of wheat by three states in the year 2002, 2003 and 2004 more or less than that in the year 2005, 2006 and 2007?

A. 2.5%

B. 2.6%

C. 1.9%

D. 1.09%

E. None of these

 

Q. 69 What was the total production of wheat by these three states in the year 2007? (in quintals)

A. 9900

B. 9700

C. 9989

D. 8800

E. None of these

 

Questions: 70 – 75

Directions (70-75) :

In the following bar diagram the number of engineers employed in various companies has been given. Study the bar diagram carefully to answer the questions.

 

Q. 70 The number of post graduate engineers employed in the company W is what percent of the total engineers employed in that company?

A. 33+1/3

B. 30+1/3

C. 25+1/3

D. 36+1/3

E. None of these

 

Q. 71 What is the average number of junior engineers employed in all the companies?

A. 150

B. 170

C. 160

D. 180

E. 190

 

Q. 72 What is the difference between the average number of junior engineers and assistant engineers taking all the companies together?

A. 18

B. 15

C. 10

D. 22

E. 25

 

Q. 73 If the number of assistant engineers employed in all the companies be increased by 37% and the number of post graduate engineers employed in all the companies be decreased by 20%, by what percent will the number of assistant engineers be less than that of post graduate engineers?

A. 5.6%

B. 7.8%

C. 8%

D. 9.3%

E. None of these

 

Q. 74 If the number of all the engineers in the company V, company X and company Y be increased by 30%, 35% and 40% respectively, what will be the overall percentage increase in the number of all the engineers of all the companies taken together?

A. 20%

B. 22%

C. 24%

D. 25%

E. None of these

 

Q. 75 What is the ratio between the number of assistant engineers employed in company V and company X?

A. 3:5

B. 3:4

C. 2:3

D. 3:2

E. None of these

 

Questions: 76 – 81

Directions (76-81) :

In the following pie charts the percentage of employees of a company working in 8 different countries had been given. Study these pie charts carefully to answer the questions.

 

Q. 76 What is the ratio between male employees working in the country I and country II

Respectively?

A. 3:4

B. 5:4

C. 5:3

D. 3:2

E. 2:3

 

Q. 77 What is the ratio between the number of male employees and female employees in country II?

A. 70:53

B. 70:31

C. 70:59

D. 53:70

E. None of these

 

Q. 78 What is the approximate average number of male employees in countries I, II and III?

A. 9670

B. 6970

C. 6907

D. 6977

E. None of these

 

Q. 79 What is the average number of female employees in the countries IV and VII?

A. 1370

B. 1070

C. 1570

D. 1470

E. None of these

 

Q. 80 If an increase of 40% is made in the average number of female employees, working countries III, IV and V, then their resulting average number will be what percent of the average number of female employees?

A. 83%

B. 85%

C. 65%

D. 68%

E. 69%

 

Q. 81 By what percent is the total number of employees in countries V, VI and VII more than the number of male employees working in the countries II, III and IV?

A. 50.2%

B. 53.6%

C. 55%

D. 48%

E. 49%

 

Questions: 82 – 87

Directions (82-87) :

In the following table, information regarding publishing books by different publishers has been given. Study the table carefully and answer the questions.

 

Q. 82 What is the difference between the number of academic books published by publishing houses M and P?

A. 450

B. 640

C. 540

D. 504

E. None of these

 

Q. 83 How many books were given to each distributor by publisher Q if each publisher gets equal number of books?

A. 1806

B. 1068

C. 1608

D. 1308

E. None of these

 

Q. 84 What is the average number of non-academic books published by publishers R and S?

A. 18750

B. 18850

C. 19950

D. 18950

E. 19990

 

Q. 85 What is the total number of books distributed by publishers O and Q?

A. 26702

B. 27324

C. 55026

D. 54026

E. None of these

 

Q. 86 If the total number of books published by publishers P, Q and R is increased by 30% and the total number of books published by remaining publishers decrease by 20%, what will be the new average of books published by an the publishers?

A. 33418

B. 33318

C. 32518

D. 33618

E. None of these

 

Q. 87 The ratio of the number of books published by publishers P and Q is

A. 156:139

B. 156:169

C. 3:5

D. 5:17

E. None of these

 

Questions: 88 – 95

Directions (88-95) :

In the following table the number of candidates appeared and qualified for different posts in an institute during various years has been given. Read the table carefully and answer the questions.

 

Q. 88 What is the approximate average number of candidates who qualified for all the posts in the year 2002?

A. 300

B. 280

C. 290

D. 310

E. 314

 

Q. 89 What is the approximate average number of candidates who appeared for all the posts in the year 2006?

A. 366

B. 389

C. 386

D. 375

E. 390

 

Q. 90 The difference between the number of candidates who appeared and qualified for the post D in the year 2001, 2002 and 2003 is

A. 615

B. 605

C. 601

D. 515

E. 505

 

Q. 91 What approximate percentage of candidates qualified for the post of A in the year 2003?

A. 73%

B. 70%

C. 68%

D. 69%

E. 75%

 

Q. 92 The respective ratio between the number of candidates who qualified for the posts of A and D in the year 2002 is

A. 67:38

B. 38:63

C. 38:67

D. 19:31

E. None of these

 

Q. 93 In which year maximum numbers of candidates were selected for the post of A?

A. 2003

B. 2004

C. 2005

D. 2006

E. 2002

 

Q. 94 In which year minimum numbers of candidates were qualified for the post of F?

A. 2002

B. 2003

C. 2004

D. 2006

E. 2005

 

Q. 95 The average number of candidates who qualified for all the posts in the year 2004 is

A. 300

B. 315

C. 328

D. 345

E. 360

 

Questions: 96 – 100

Directions (96-100) :

Study the pie charts carefully to answer the following questions.

 

Q. 96 Number of girls enrolled in dancing form what percent of total number of students in the school? (rounded off to two digits after decimal)

A. 12.35

B. 14.12

C. 11.67

D. 10.08

E. None of these

 

Q. 97 How many boys are enrolled in singing and craft together?

A. 505

B. 610

C. 485

D. 420

E. None of these

 

Q. 98 What is the respective ratio of number of girls enrolled in swimming to the number of boys enrolled in swimming?

A. 47:49

B. 23:29

C. 29:23

D. 49:47

E. None of these

 

Q. 99 What is the total number of girls enrolled in swimming and drawing together?

A. 480

B. 525

C. 505

D. 495

E. None of these

 

Q. 100 What is the approximate percentage of boys in the school?

A. 34

B. 56

C. 28

D. 50

E. None of these

 

 

Answer Sheet
Question 1 2 3 4 5 6 7 8 9 10
Answer C A D B C D A E B D
Question 11 12 13 14 15 16 17 18 19 20
Answer A D E A B C C C B E
Question 21 22 23 24 25 26 27 28 29 30
Answer C D A E B D A B E A
Question 31 32 33 34 35 36 37 38 39 40
Answer C D B A B D A C E A
Question 41 42 43 44 45 46 47 48 49 50
Answer A D B E B D D A D E
Question 51 52 53 54 55 56 57 58 59 60
Answer A B C D A B C A B C
Question 61 62 63 64 65 66 67 68 69 70
Answer A A C A B C B D A A
Question 71 72 73 74 75 76 77 78 79 80
Answer B C D B B B C C D A
Question 81 82 83 84 85 86 87 88 89 90
Answer B C B C D B B A C D
Question 91 92 93 94 95 96 97 98 99 100
Answer A B D C C C A D B E
×

Hello!

Click one of our representatives below to chat on WhatsApp or send us an email to info@vidhyarthidarpan.com

×